Slot System
Featured Buckets
Featured Buckets Admin

COVID-19 mythconceptions

Article Type
Changed
Thu, 08/26/2021 - 16:13

his month, I would like to touch on a few COVID-19 topics that have received much publicity, with some messages about them having been confusing.

Paauw_Doug_SEATTLE_2019_web2.jpg
Dr. Douglas S. Paauw

Let’s start with a case:

A 37-year-old woman is seen in clinic for a 5-day history of cough, fever, chest tightness, and onset of dyspnea on the day of her office visit.

An exam reveals her blood pressure is 100/60 mm Hg, her pulse is 100 beats per minute, her temperature is 38.7° C, her oxygen saturation is 93%, and her respiratory rate is 20 breaths per minute.

Auscultation of the chest revealed bilateral wheezing and rhonchi. A nasopharyngeal swab is sent for COVID-19 and is negative; she also tests negative for influenza.

Her hemoglobin level is 13 g/dL, hematocrit was 39%, platelet count was 155,000 per mcL of blood, and D-dimer level was 8.4 mcg/mL (normal is less than 0.4 mcg/mL.) Her white blood cell count was 6,000 per mcL of blood (neutrophils, 4,900; lymphocytes, 800; basophils, 200). Her chest x-ray showed bilateral lower lobe infiltrates.
 

What do you recommend?

A. Begin azithromycin plus ceftriaxone

B. Begin azithromycin

C. Begin oseltamivir

D. Obtain chest CT

E. Repeat COVID-19 test

With the massive amount of information coming out every day on COVID-19, it is hard to keep up with all of it, and sort out accurate, reviewed studies. We are in a position where we need to take in what we can and assess the best data available.

In the case above, I think choices D or E would make sense. This patient very likely has COVID-19 based on clinical symptoms and lab parameters. The negative COVID-19 test gives us pause, but several studies show that false negative tests are not uncommon.

Long et al. reported on 36 patients who had received both chest CT and real-time reverse transcription polymerase chain reaction (rRT-PCR) for COVID-19.1 All were eventually diagnosed with COVID-19 pneumonia. The CT scan had a very high sensitivity (35/36) of 97.2%, whereas the rRT-PCR had a lower sensitivity (30/36) of 83%. All six of the patients with a negative COVID-19 test initially were positive on repeat testing (three on the second test, three on the third test).

There are concerns about what the sensitivity of the rRT-PCR tests being run in the United States are. At this point, I think that, when the pretest probability of COVID-19 infection is very high based on local epidemiology and clinical symptoms, a negative COVID rRT-PCR does not eliminate the diagnosis. In many cases, COVID-19 may still be the most likely diagnosis.

Early in the pandemic, the symptoms that were emphasized were fever, cough, and dyspnea. Those were all crucial symptoms for a disease that causes pneumonia. GI symptoms were initially deemphasized. In an early study released from Wuhan, China, only about 5% of COVID-19 patients had nausea or diarrhea.2 In a study of 305 patients focused on gastrointestinal symptoms, half of the patients had diarrhea, half had anorexia and 30% had nausea.3 In a small series of nine patients who presented with only GI symptoms, four of these patients never developed fever or pulmonary symptoms.3

On March 14, the French health minister, Olivier Véran, tweeted that “taking anti-inflammatory drugs (ibuprofen, cortisone ...) could be an aggravating factor for the infection. If you have a fever, take paracetamol.” This was picked up by many news services, and soon became standard recommendations, despite no data.

There is reason for concern for NSAIDs, as regular NSAID use has been tied to more complications in patients with respiratory tract infections.4 I have never been a proponent of regular NSAID use in patients who are infected, because the likelihood of toxicity is elevated in patients who are volume depleted or under physiologic stress. But at this time, there is no evidence on problems with episodic NSAID use in patients with COVID-19.

Another widely disseminated decree was that patients with COVID-19 should not use ACE inhibitors and angiotensin II receptor blockers (ARBs). COVID-19 binds to their target cells through ACE2, which is expressed by epithelial cells of the lung, intestine and kidney. Patients who are treated with ACE inhibitors and ARBs have been shown to have more ACE2 expression.

In a letter to the editor by Fang et al. published in Lancet Respiratory Medicine, the authors raised the question of whether patients might be better served to be switched from ACE inhibitors and ARBs to calcium-channel blockers for the treatment of hypertension.5 A small study by Meng et al. looked at outcomes of patients on these drugs who had COVID-19 infection.6 They looked at 417 patients admitted to a hospital in China with COVID-19 infection. A total of 42 patients were on medications for hypertension. Group 1 were patients on ACE inhibitors/ARBs (17 patients) and group 2 were patients on other antihypertensives (25 patients). During hospitalization 12 patients (48%) in group 2 were categorized as having severe disease and 1 patient died. In group 1 (the ACE inhibitor/ARB–treated patients) only four (23%) were categorized as having severe disease, and no patients in this group died.

Vaduganathan et al. published a special report in the New England Journal of Medicine strongly arguing the point that “[u]ntil further data are available, we think that [renin-angiotensin-aldosterone system] inhibitors should be continued in patients in otherwise stable condition who are at risk for, being evaluated for, or with COVID-19”.7 This position is supported by the American Heart Association, the American College of Cardiology, the American College of Physicians, and 11 other medical organizations.
 

Take-home messages

  • Testing isn’t perfect – if you have strong suspicion for COVID-19 disease, retest.
  • GI symptoms appear to be common, and rarely may be the only symptoms initially.
  • NSAIDs are always risky in really sick patients, but data specific to COVID-19 is lacking.
  • ACE inhibitors/ARBs should not be avoided in patients with COVID-19.

Dr. Paauw is professor of medicine in the division of general internal medicine at the University of Washington, Seattle, and he serves as third-year medical student clerkship director at the University of Washington. He is a member of the editorial advisory board of Internal Medicine News. Dr. Paauw has no conflicts to disclose. Contact Dr. Paauw at imnews@mdedge.com.

References

1. Long C et al. Diagnosis of the Coronavirus disease (COVID-19): rRT-PCR or CT? Eur J Radiol. 2020 Mar 25;126:108961.

2. Zhou F et al. Clinical course and risk factors for mortality of adult inpatients with COVID-19 in Wuhan, China: A retrospective cohort study. Lancet. 2020 Mar 28;395(10229):1054-62.

3. Tian Y et al. Review article: Gastrointestinal features in COVID-19 and the possibility of faecal transmission. Aliment Pharmacol Ther. 2020;00:1–9.

4. Voiriot G et al. Risks related to the use of nonsteroidal anti-inflammatory drugs in community-acquired pneumonia in adult and pediatric patients. J Clin Med. 2019;8:E786.

5. Fang L et al. Are patients with hypertension and diabetes mellitus at increased risk for COVID-19 infection? Lancet Respir Med. 2020 Mar 11. doi:10.1016/S2213-2600(20)30116-8.

6. Meng J et al. Renin-angiotensin system inhibitors improve the clinical outcomes of COVID-19 patients with hypertension. Renin-angiotensin system inhibitors improve the clinical outcomes of COVID-19 patients with hypertension. Emerg Microbes Infect. 2020 Dec;9(1):757-60.

7. Vaduganathan M et al. Renin-angiotensin-aldosterone system inhibitors in patients with COVID-19. N Engl J Med. 2020 Mar 30. doi: 10.1056/NEJMsr2005760.

Publications
Topics
Sections

his month, I would like to touch on a few COVID-19 topics that have received much publicity, with some messages about them having been confusing.

Paauw_Doug_SEATTLE_2019_web2.jpg
Dr. Douglas S. Paauw

Let’s start with a case:

A 37-year-old woman is seen in clinic for a 5-day history of cough, fever, chest tightness, and onset of dyspnea on the day of her office visit.

An exam reveals her blood pressure is 100/60 mm Hg, her pulse is 100 beats per minute, her temperature is 38.7° C, her oxygen saturation is 93%, and her respiratory rate is 20 breaths per minute.

Auscultation of the chest revealed bilateral wheezing and rhonchi. A nasopharyngeal swab is sent for COVID-19 and is negative; she also tests negative for influenza.

Her hemoglobin level is 13 g/dL, hematocrit was 39%, platelet count was 155,000 per mcL of blood, and D-dimer level was 8.4 mcg/mL (normal is less than 0.4 mcg/mL.) Her white blood cell count was 6,000 per mcL of blood (neutrophils, 4,900; lymphocytes, 800; basophils, 200). Her chest x-ray showed bilateral lower lobe infiltrates.
 

What do you recommend?

A. Begin azithromycin plus ceftriaxone

B. Begin azithromycin

C. Begin oseltamivir

D. Obtain chest CT

E. Repeat COVID-19 test

With the massive amount of information coming out every day on COVID-19, it is hard to keep up with all of it, and sort out accurate, reviewed studies. We are in a position where we need to take in what we can and assess the best data available.

In the case above, I think choices D or E would make sense. This patient very likely has COVID-19 based on clinical symptoms and lab parameters. The negative COVID-19 test gives us pause, but several studies show that false negative tests are not uncommon.

Long et al. reported on 36 patients who had received both chest CT and real-time reverse transcription polymerase chain reaction (rRT-PCR) for COVID-19.1 All were eventually diagnosed with COVID-19 pneumonia. The CT scan had a very high sensitivity (35/36) of 97.2%, whereas the rRT-PCR had a lower sensitivity (30/36) of 83%. All six of the patients with a negative COVID-19 test initially were positive on repeat testing (three on the second test, three on the third test).

There are concerns about what the sensitivity of the rRT-PCR tests being run in the United States are. At this point, I think that, when the pretest probability of COVID-19 infection is very high based on local epidemiology and clinical symptoms, a negative COVID rRT-PCR does not eliminate the diagnosis. In many cases, COVID-19 may still be the most likely diagnosis.

Early in the pandemic, the symptoms that were emphasized were fever, cough, and dyspnea. Those were all crucial symptoms for a disease that causes pneumonia. GI symptoms were initially deemphasized. In an early study released from Wuhan, China, only about 5% of COVID-19 patients had nausea or diarrhea.2 In a study of 305 patients focused on gastrointestinal symptoms, half of the patients had diarrhea, half had anorexia and 30% had nausea.3 In a small series of nine patients who presented with only GI symptoms, four of these patients never developed fever or pulmonary symptoms.3

On March 14, the French health minister, Olivier Véran, tweeted that “taking anti-inflammatory drugs (ibuprofen, cortisone ...) could be an aggravating factor for the infection. If you have a fever, take paracetamol.” This was picked up by many news services, and soon became standard recommendations, despite no data.

There is reason for concern for NSAIDs, as regular NSAID use has been tied to more complications in patients with respiratory tract infections.4 I have never been a proponent of regular NSAID use in patients who are infected, because the likelihood of toxicity is elevated in patients who are volume depleted or under physiologic stress. But at this time, there is no evidence on problems with episodic NSAID use in patients with COVID-19.

Another widely disseminated decree was that patients with COVID-19 should not use ACE inhibitors and angiotensin II receptor blockers (ARBs). COVID-19 binds to their target cells through ACE2, which is expressed by epithelial cells of the lung, intestine and kidney. Patients who are treated with ACE inhibitors and ARBs have been shown to have more ACE2 expression.

In a letter to the editor by Fang et al. published in Lancet Respiratory Medicine, the authors raised the question of whether patients might be better served to be switched from ACE inhibitors and ARBs to calcium-channel blockers for the treatment of hypertension.5 A small study by Meng et al. looked at outcomes of patients on these drugs who had COVID-19 infection.6 They looked at 417 patients admitted to a hospital in China with COVID-19 infection. A total of 42 patients were on medications for hypertension. Group 1 were patients on ACE inhibitors/ARBs (17 patients) and group 2 were patients on other antihypertensives (25 patients). During hospitalization 12 patients (48%) in group 2 were categorized as having severe disease and 1 patient died. In group 1 (the ACE inhibitor/ARB–treated patients) only four (23%) were categorized as having severe disease, and no patients in this group died.

Vaduganathan et al. published a special report in the New England Journal of Medicine strongly arguing the point that “[u]ntil further data are available, we think that [renin-angiotensin-aldosterone system] inhibitors should be continued in patients in otherwise stable condition who are at risk for, being evaluated for, or with COVID-19”.7 This position is supported by the American Heart Association, the American College of Cardiology, the American College of Physicians, and 11 other medical organizations.
 

Take-home messages

  • Testing isn’t perfect – if you have strong suspicion for COVID-19 disease, retest.
  • GI symptoms appear to be common, and rarely may be the only symptoms initially.
  • NSAIDs are always risky in really sick patients, but data specific to COVID-19 is lacking.
  • ACE inhibitors/ARBs should not be avoided in patients with COVID-19.

Dr. Paauw is professor of medicine in the division of general internal medicine at the University of Washington, Seattle, and he serves as third-year medical student clerkship director at the University of Washington. He is a member of the editorial advisory board of Internal Medicine News. Dr. Paauw has no conflicts to disclose. Contact Dr. Paauw at imnews@mdedge.com.

References

1. Long C et al. Diagnosis of the Coronavirus disease (COVID-19): rRT-PCR or CT? Eur J Radiol. 2020 Mar 25;126:108961.

2. Zhou F et al. Clinical course and risk factors for mortality of adult inpatients with COVID-19 in Wuhan, China: A retrospective cohort study. Lancet. 2020 Mar 28;395(10229):1054-62.

3. Tian Y et al. Review article: Gastrointestinal features in COVID-19 and the possibility of faecal transmission. Aliment Pharmacol Ther. 2020;00:1–9.

4. Voiriot G et al. Risks related to the use of nonsteroidal anti-inflammatory drugs in community-acquired pneumonia in adult and pediatric patients. J Clin Med. 2019;8:E786.

5. Fang L et al. Are patients with hypertension and diabetes mellitus at increased risk for COVID-19 infection? Lancet Respir Med. 2020 Mar 11. doi:10.1016/S2213-2600(20)30116-8.

6. Meng J et al. Renin-angiotensin system inhibitors improve the clinical outcomes of COVID-19 patients with hypertension. Renin-angiotensin system inhibitors improve the clinical outcomes of COVID-19 patients with hypertension. Emerg Microbes Infect. 2020 Dec;9(1):757-60.

7. Vaduganathan M et al. Renin-angiotensin-aldosterone system inhibitors in patients with COVID-19. N Engl J Med. 2020 Mar 30. doi: 10.1056/NEJMsr2005760.

his month, I would like to touch on a few COVID-19 topics that have received much publicity, with some messages about them having been confusing.

Paauw_Doug_SEATTLE_2019_web2.jpg
Dr. Douglas S. Paauw

Let’s start with a case:

A 37-year-old woman is seen in clinic for a 5-day history of cough, fever, chest tightness, and onset of dyspnea on the day of her office visit.

An exam reveals her blood pressure is 100/60 mm Hg, her pulse is 100 beats per minute, her temperature is 38.7° C, her oxygen saturation is 93%, and her respiratory rate is 20 breaths per minute.

Auscultation of the chest revealed bilateral wheezing and rhonchi. A nasopharyngeal swab is sent for COVID-19 and is negative; she also tests negative for influenza.

Her hemoglobin level is 13 g/dL, hematocrit was 39%, platelet count was 155,000 per mcL of blood, and D-dimer level was 8.4 mcg/mL (normal is less than 0.4 mcg/mL.) Her white blood cell count was 6,000 per mcL of blood (neutrophils, 4,900; lymphocytes, 800; basophils, 200). Her chest x-ray showed bilateral lower lobe infiltrates.
 

What do you recommend?

A. Begin azithromycin plus ceftriaxone

B. Begin azithromycin

C. Begin oseltamivir

D. Obtain chest CT

E. Repeat COVID-19 test

With the massive amount of information coming out every day on COVID-19, it is hard to keep up with all of it, and sort out accurate, reviewed studies. We are in a position where we need to take in what we can and assess the best data available.

In the case above, I think choices D or E would make sense. This patient very likely has COVID-19 based on clinical symptoms and lab parameters. The negative COVID-19 test gives us pause, but several studies show that false negative tests are not uncommon.

Long et al. reported on 36 patients who had received both chest CT and real-time reverse transcription polymerase chain reaction (rRT-PCR) for COVID-19.1 All were eventually diagnosed with COVID-19 pneumonia. The CT scan had a very high sensitivity (35/36) of 97.2%, whereas the rRT-PCR had a lower sensitivity (30/36) of 83%. All six of the patients with a negative COVID-19 test initially were positive on repeat testing (three on the second test, three on the third test).

There are concerns about what the sensitivity of the rRT-PCR tests being run in the United States are. At this point, I think that, when the pretest probability of COVID-19 infection is very high based on local epidemiology and clinical symptoms, a negative COVID rRT-PCR does not eliminate the diagnosis. In many cases, COVID-19 may still be the most likely diagnosis.

Early in the pandemic, the symptoms that were emphasized were fever, cough, and dyspnea. Those were all crucial symptoms for a disease that causes pneumonia. GI symptoms were initially deemphasized. In an early study released from Wuhan, China, only about 5% of COVID-19 patients had nausea or diarrhea.2 In a study of 305 patients focused on gastrointestinal symptoms, half of the patients had diarrhea, half had anorexia and 30% had nausea.3 In a small series of nine patients who presented with only GI symptoms, four of these patients never developed fever or pulmonary symptoms.3

On March 14, the French health minister, Olivier Véran, tweeted that “taking anti-inflammatory drugs (ibuprofen, cortisone ...) could be an aggravating factor for the infection. If you have a fever, take paracetamol.” This was picked up by many news services, and soon became standard recommendations, despite no data.

There is reason for concern for NSAIDs, as regular NSAID use has been tied to more complications in patients with respiratory tract infections.4 I have never been a proponent of regular NSAID use in patients who are infected, because the likelihood of toxicity is elevated in patients who are volume depleted or under physiologic stress. But at this time, there is no evidence on problems with episodic NSAID use in patients with COVID-19.

Another widely disseminated decree was that patients with COVID-19 should not use ACE inhibitors and angiotensin II receptor blockers (ARBs). COVID-19 binds to their target cells through ACE2, which is expressed by epithelial cells of the lung, intestine and kidney. Patients who are treated with ACE inhibitors and ARBs have been shown to have more ACE2 expression.

In a letter to the editor by Fang et al. published in Lancet Respiratory Medicine, the authors raised the question of whether patients might be better served to be switched from ACE inhibitors and ARBs to calcium-channel blockers for the treatment of hypertension.5 A small study by Meng et al. looked at outcomes of patients on these drugs who had COVID-19 infection.6 They looked at 417 patients admitted to a hospital in China with COVID-19 infection. A total of 42 patients were on medications for hypertension. Group 1 were patients on ACE inhibitors/ARBs (17 patients) and group 2 were patients on other antihypertensives (25 patients). During hospitalization 12 patients (48%) in group 2 were categorized as having severe disease and 1 patient died. In group 1 (the ACE inhibitor/ARB–treated patients) only four (23%) were categorized as having severe disease, and no patients in this group died.

Vaduganathan et al. published a special report in the New England Journal of Medicine strongly arguing the point that “[u]ntil further data are available, we think that [renin-angiotensin-aldosterone system] inhibitors should be continued in patients in otherwise stable condition who are at risk for, being evaluated for, or with COVID-19”.7 This position is supported by the American Heart Association, the American College of Cardiology, the American College of Physicians, and 11 other medical organizations.
 

Take-home messages

  • Testing isn’t perfect – if you have strong suspicion for COVID-19 disease, retest.
  • GI symptoms appear to be common, and rarely may be the only symptoms initially.
  • NSAIDs are always risky in really sick patients, but data specific to COVID-19 is lacking.
  • ACE inhibitors/ARBs should not be avoided in patients with COVID-19.

Dr. Paauw is professor of medicine in the division of general internal medicine at the University of Washington, Seattle, and he serves as third-year medical student clerkship director at the University of Washington. He is a member of the editorial advisory board of Internal Medicine News. Dr. Paauw has no conflicts to disclose. Contact Dr. Paauw at imnews@mdedge.com.

References

1. Long C et al. Diagnosis of the Coronavirus disease (COVID-19): rRT-PCR or CT? Eur J Radiol. 2020 Mar 25;126:108961.

2. Zhou F et al. Clinical course and risk factors for mortality of adult inpatients with COVID-19 in Wuhan, China: A retrospective cohort study. Lancet. 2020 Mar 28;395(10229):1054-62.

3. Tian Y et al. Review article: Gastrointestinal features in COVID-19 and the possibility of faecal transmission. Aliment Pharmacol Ther. 2020;00:1–9.

4. Voiriot G et al. Risks related to the use of nonsteroidal anti-inflammatory drugs in community-acquired pneumonia in adult and pediatric patients. J Clin Med. 2019;8:E786.

5. Fang L et al. Are patients with hypertension and diabetes mellitus at increased risk for COVID-19 infection? Lancet Respir Med. 2020 Mar 11. doi:10.1016/S2213-2600(20)30116-8.

6. Meng J et al. Renin-angiotensin system inhibitors improve the clinical outcomes of COVID-19 patients with hypertension. Renin-angiotensin system inhibitors improve the clinical outcomes of COVID-19 patients with hypertension. Emerg Microbes Infect. 2020 Dec;9(1):757-60.

7. Vaduganathan M et al. Renin-angiotensin-aldosterone system inhibitors in patients with COVID-19. N Engl J Med. 2020 Mar 30. doi: 10.1056/NEJMsr2005760.

Publications
Publications
Topics
Article Type
Sections
Disallow All Ads
Content Gating
No Gating (article Unlocked/Free)
Alternative CME
Disqus Comments
Default
Use ProPublica
Hide sidebar & use full width
render the right sidebar.

Can this patient get IV contrast?

Article Type
Changed
Mon, 03/02/2020 - 21:52

A 59-year-old man is admitted with abdominal pain. He has a history of pancreatitis. A contrast CT scan is ordered. He reports a history of severe shellfish allergy when the radiology tech checks him in for the procedure. You are paged regarding what to do:

A) Continue with scan as ordered.

B) Switch to MRI scan.

C) Switch to MRI scan with gadolinium.

D) Continue with CT with contrast, give dose of Solu-Medrol.

E) Continue with CT with contrast give IV diphenhydramine.
 

The correct answer here is A, This patient can receive his scan and receive contrast as ordered.

For many years, patients have been asked about shellfish allergy as a proxy for having increased risk when receiving iodine containing contrast. The mistaken thought was that shellfish contains iodine, so allergy to shellfish was likely to portend allergy to iodine.

Paauw_Doug_SEATTLE_2019_web.jpg
Dr. Douglas S. Paauw

Allergy to shellfish is caused by individual proteins that are definitely not in iodine-containing contrast.1 Beaty et al. studied the prevalence of the belief that allergy to shellfish is tied to iodine allergy in a survey given to 231 faculty radiologists and interventional cardiologists.2 Almost 70% responded that they inquire about seafood allergy before procedures that require iodine contrast, and 37% reported they would withhold the contrast or premedicate patients if they had a seafood allergy.

In a more recent study, Westermann-Clark and colleagues surveyed 252 health professionals before and after an educational intervention to dispel the myth of shellfish allergy and iodinated contrast reactions.3 Before the intervention, 66% of participants felt it was important to ask about shellfish allergies and 93% felt it was important to ask about iodine allergies; 26% responded that they would withhold iodinated contrast material in patients with a shellfish allergy, and 56% would withhold in patients with an iodine allergy. A total of 62% reported they would premedicate patients with a shellfish allergy and 75% would premedicate patients with an iodine allergy. The numbers declined dramatically after the educational intervention.

Patients who have seafood allergy have a higher rate of reactions to iodinated contrast, but not at a higher rate than do patients with other food allergies or asthma.4 Most radiology departments do not screen for other food allergies despite the fact these allergies have the same increased risk as for patients with a seafood/shellfish allergy. These patients are more allergic, and in general, are more likely to have reactions. The American Academy of Allergy, Asthma, and Immunology recommends not routinely ordering low- or iso-osmolar radiocontrast media or pretreating with either antihistamines or steroids in patients with a history of seafood allergy.5

Plate_of_seafood_web.jpg


There is no evidence that iodine causes allergic reactions. It makes sense that iodine does not cause allergic reactions, as it is an essential component in the human body, in thyroid hormone and in amino acids.6 Patients with dermatitis following topical application of iodine preparations such as povidone-iodide are not reacting to the iodine.

Van Ketel and van den Berg patch-tested patients with a history of dermatitis after exposure to povidone-iodine.7 All patients reacted to patch testing with povidone-iodine, but none reacted to direct testing to iodine (0/5 with patch testing of potassium iodide and 0/3 with testing with iodine tincture).


Take home points:

  • It is unnecessary and unhelpful to ask patients about seafood allergies before ordering radiologic studies involving contrast.
  • Iodine allergy does not exist.

Dr. Paauw is professor of medicine in the division of general internal medicine at the University of Washington, Seattle, and he serves as third-year medical student clerkship director at the University of Washington. Contact Dr. Paauw at dpaauw@uw.edu.

References

1. Narayan AK et al. Avoiding contrast-enhanced computed tomography scans in patients with shellfish allergies. J Hosp Med. 2016 Jun;11(6):435-7.

2. Beaty AD et al. Seafood allergy and radiocontrast media: Are physicians propagating a myth? Am J Med. 2008 Feb;121(2):158.e1-4.

3. Westermann-Clark E et al. Debunking myths about “allergy” to radiocontrast media in an academic institution. Postgrad Med. 2015 Apr;127(3):295-300.

4. Coakley FV and DM Panicek. Iodine allergy: An oyster without a pearl? AJR Am J Roentgenol. 1997 Oct;169(4):951-2.

5. American Academy of Allergy, Asthma & Immunology recommendations on low- or iso-osmolar radiocontrast media.

6. Schabelman E and M Witting. The relationship of radiocontrast, iodine, and seafood allergies: A medical myth exposed. J Emerg Med. 2010 Nov;39(5):701-7.

7. van Ketel WG and WH van den Berg. Sensitization to povidone-iodine. Dermatol Clin. 1990 Jan;8(1):107-9.

Publications
Topics
Sections

A 59-year-old man is admitted with abdominal pain. He has a history of pancreatitis. A contrast CT scan is ordered. He reports a history of severe shellfish allergy when the radiology tech checks him in for the procedure. You are paged regarding what to do:

A) Continue with scan as ordered.

B) Switch to MRI scan.

C) Switch to MRI scan with gadolinium.

D) Continue with CT with contrast, give dose of Solu-Medrol.

E) Continue with CT with contrast give IV diphenhydramine.
 

The correct answer here is A, This patient can receive his scan and receive contrast as ordered.

For many years, patients have been asked about shellfish allergy as a proxy for having increased risk when receiving iodine containing contrast. The mistaken thought was that shellfish contains iodine, so allergy to shellfish was likely to portend allergy to iodine.

Paauw_Doug_SEATTLE_2019_web.jpg
Dr. Douglas S. Paauw

Allergy to shellfish is caused by individual proteins that are definitely not in iodine-containing contrast.1 Beaty et al. studied the prevalence of the belief that allergy to shellfish is tied to iodine allergy in a survey given to 231 faculty radiologists and interventional cardiologists.2 Almost 70% responded that they inquire about seafood allergy before procedures that require iodine contrast, and 37% reported they would withhold the contrast or premedicate patients if they had a seafood allergy.

In a more recent study, Westermann-Clark and colleagues surveyed 252 health professionals before and after an educational intervention to dispel the myth of shellfish allergy and iodinated contrast reactions.3 Before the intervention, 66% of participants felt it was important to ask about shellfish allergies and 93% felt it was important to ask about iodine allergies; 26% responded that they would withhold iodinated contrast material in patients with a shellfish allergy, and 56% would withhold in patients with an iodine allergy. A total of 62% reported they would premedicate patients with a shellfish allergy and 75% would premedicate patients with an iodine allergy. The numbers declined dramatically after the educational intervention.

Patients who have seafood allergy have a higher rate of reactions to iodinated contrast, but not at a higher rate than do patients with other food allergies or asthma.4 Most radiology departments do not screen for other food allergies despite the fact these allergies have the same increased risk as for patients with a seafood/shellfish allergy. These patients are more allergic, and in general, are more likely to have reactions. The American Academy of Allergy, Asthma, and Immunology recommends not routinely ordering low- or iso-osmolar radiocontrast media or pretreating with either antihistamines or steroids in patients with a history of seafood allergy.5

Plate_of_seafood_web.jpg


There is no evidence that iodine causes allergic reactions. It makes sense that iodine does not cause allergic reactions, as it is an essential component in the human body, in thyroid hormone and in amino acids.6 Patients with dermatitis following topical application of iodine preparations such as povidone-iodide are not reacting to the iodine.

Van Ketel and van den Berg patch-tested patients with a history of dermatitis after exposure to povidone-iodine.7 All patients reacted to patch testing with povidone-iodine, but none reacted to direct testing to iodine (0/5 with patch testing of potassium iodide and 0/3 with testing with iodine tincture).


Take home points:

  • It is unnecessary and unhelpful to ask patients about seafood allergies before ordering radiologic studies involving contrast.
  • Iodine allergy does not exist.

Dr. Paauw is professor of medicine in the division of general internal medicine at the University of Washington, Seattle, and he serves as third-year medical student clerkship director at the University of Washington. Contact Dr. Paauw at dpaauw@uw.edu.

References

1. Narayan AK et al. Avoiding contrast-enhanced computed tomography scans in patients with shellfish allergies. J Hosp Med. 2016 Jun;11(6):435-7.

2. Beaty AD et al. Seafood allergy and radiocontrast media: Are physicians propagating a myth? Am J Med. 2008 Feb;121(2):158.e1-4.

3. Westermann-Clark E et al. Debunking myths about “allergy” to radiocontrast media in an academic institution. Postgrad Med. 2015 Apr;127(3):295-300.

4. Coakley FV and DM Panicek. Iodine allergy: An oyster without a pearl? AJR Am J Roentgenol. 1997 Oct;169(4):951-2.

5. American Academy of Allergy, Asthma & Immunology recommendations on low- or iso-osmolar radiocontrast media.

6. Schabelman E and M Witting. The relationship of radiocontrast, iodine, and seafood allergies: A medical myth exposed. J Emerg Med. 2010 Nov;39(5):701-7.

7. van Ketel WG and WH van den Berg. Sensitization to povidone-iodine. Dermatol Clin. 1990 Jan;8(1):107-9.

A 59-year-old man is admitted with abdominal pain. He has a history of pancreatitis. A contrast CT scan is ordered. He reports a history of severe shellfish allergy when the radiology tech checks him in for the procedure. You are paged regarding what to do:

A) Continue with scan as ordered.

B) Switch to MRI scan.

C) Switch to MRI scan with gadolinium.

D) Continue with CT with contrast, give dose of Solu-Medrol.

E) Continue with CT with contrast give IV diphenhydramine.
 

The correct answer here is A, This patient can receive his scan and receive contrast as ordered.

For many years, patients have been asked about shellfish allergy as a proxy for having increased risk when receiving iodine containing contrast. The mistaken thought was that shellfish contains iodine, so allergy to shellfish was likely to portend allergy to iodine.

Paauw_Doug_SEATTLE_2019_web.jpg
Dr. Douglas S. Paauw

Allergy to shellfish is caused by individual proteins that are definitely not in iodine-containing contrast.1 Beaty et al. studied the prevalence of the belief that allergy to shellfish is tied to iodine allergy in a survey given to 231 faculty radiologists and interventional cardiologists.2 Almost 70% responded that they inquire about seafood allergy before procedures that require iodine contrast, and 37% reported they would withhold the contrast or premedicate patients if they had a seafood allergy.

In a more recent study, Westermann-Clark and colleagues surveyed 252 health professionals before and after an educational intervention to dispel the myth of shellfish allergy and iodinated contrast reactions.3 Before the intervention, 66% of participants felt it was important to ask about shellfish allergies and 93% felt it was important to ask about iodine allergies; 26% responded that they would withhold iodinated contrast material in patients with a shellfish allergy, and 56% would withhold in patients with an iodine allergy. A total of 62% reported they would premedicate patients with a shellfish allergy and 75% would premedicate patients with an iodine allergy. The numbers declined dramatically after the educational intervention.

Patients who have seafood allergy have a higher rate of reactions to iodinated contrast, but not at a higher rate than do patients with other food allergies or asthma.4 Most radiology departments do not screen for other food allergies despite the fact these allergies have the same increased risk as for patients with a seafood/shellfish allergy. These patients are more allergic, and in general, are more likely to have reactions. The American Academy of Allergy, Asthma, and Immunology recommends not routinely ordering low- or iso-osmolar radiocontrast media or pretreating with either antihistamines or steroids in patients with a history of seafood allergy.5

Plate_of_seafood_web.jpg


There is no evidence that iodine causes allergic reactions. It makes sense that iodine does not cause allergic reactions, as it is an essential component in the human body, in thyroid hormone and in amino acids.6 Patients with dermatitis following topical application of iodine preparations such as povidone-iodide are not reacting to the iodine.

Van Ketel and van den Berg patch-tested patients with a history of dermatitis after exposure to povidone-iodine.7 All patients reacted to patch testing with povidone-iodine, but none reacted to direct testing to iodine (0/5 with patch testing of potassium iodide and 0/3 with testing with iodine tincture).


Take home points:

  • It is unnecessary and unhelpful to ask patients about seafood allergies before ordering radiologic studies involving contrast.
  • Iodine allergy does not exist.

Dr. Paauw is professor of medicine in the division of general internal medicine at the University of Washington, Seattle, and he serves as third-year medical student clerkship director at the University of Washington. Contact Dr. Paauw at dpaauw@uw.edu.

References

1. Narayan AK et al. Avoiding contrast-enhanced computed tomography scans in patients with shellfish allergies. J Hosp Med. 2016 Jun;11(6):435-7.

2. Beaty AD et al. Seafood allergy and radiocontrast media: Are physicians propagating a myth? Am J Med. 2008 Feb;121(2):158.e1-4.

3. Westermann-Clark E et al. Debunking myths about “allergy” to radiocontrast media in an academic institution. Postgrad Med. 2015 Apr;127(3):295-300.

4. Coakley FV and DM Panicek. Iodine allergy: An oyster without a pearl? AJR Am J Roentgenol. 1997 Oct;169(4):951-2.

5. American Academy of Allergy, Asthma & Immunology recommendations on low- or iso-osmolar radiocontrast media.

6. Schabelman E and M Witting. The relationship of radiocontrast, iodine, and seafood allergies: A medical myth exposed. J Emerg Med. 2010 Nov;39(5):701-7.

7. van Ketel WG and WH van den Berg. Sensitization to povidone-iodine. Dermatol Clin. 1990 Jan;8(1):107-9.

Publications
Publications
Topics
Article Type
Sections
Disallow All Ads
Content Gating
No Gating (article Unlocked/Free)
Alternative CME
Disqus Comments
Default
Use ProPublica
Hide sidebar & use full width
render the right sidebar.

IV fluid and narcotics for renal colic

Article Type
Changed
Fri, 01/18/2019 - 18:01

A 40-year-old man presents with severe right flank pain for 1 hour. He has had this in the past when he passed a kidney stone. Urinalysis shows greater than 100 red blood cells per high power field (HPF). CT shows a 6-mm stone in the left ureter.

What do you recommend for therapy?

A. IV ketorolac and IV fluids.

B. IV morphine and IV fluids.

C. IV morphine.

D. IV ketorolac.

This is a common scenario, especially in emergency department settings and acute care clinics. Patients arrive in severe pain because of renal colic from kidney stones. Standard teaching that I received many years ago was that this patient should receive IV fluid to “help float the stone out” and narcotic pain medications to treat the severe pain the patient was in.

Paauw_Douglas_S_WA_web.jpg
Dr. Douglas S. Paauw

Is there good evidence that this is the best therapy?

There are scant data on the practice of IV fluid for treatment of renal stone passage. W. Patrick Springhart, MD, and his colleagues studied 43 patients who presented to the ED for treatment of renal colic.1 All patients had CT evaluation for stones and received intravenous analgesia. Twenty patients were randomized to receive 2 L of normal saline over 2 hours, and 23 patients received minimal IV saline (20 mL/hour). There were no differences between the two groups in pain scores, narcotic requirements, or stone passage rates.

In an older study, Tom-Harald Edna, PhD, and colleagues studied 60 patients with ureteral colic, randomizing them to receive either no fluid or 3 L of IV fluid over 6 hours.2 There was no significant difference in pain between treatment groups.

A Cochrane analysis in 2012 concluded that there was no reliable evidence to support the use of high-volume fluid therapy in the treatment of acute ureteral colic.3

Standard treatment of pain for renal colic has been to use narcotics. In a randomized, double-blind trial comparing ketorolac and meperidine, William Cordell, MD, and his colleagues found that pain relief was superior in ketorolac-treated patients. Seventy-five percent of ketorolac patients had a 50% reduction in pain scores versus only 23% of the patients who received meperidine (P less than .001).4

Anna Holdgate and Tamara Pollock reviewed 20 studies that evaluated NSAIDs and narcotics for acute renal colic. They concluded that patients treated with NSAIDs had greater pain relief with less vomiting than did patients treated with narcotics.5

In the past decade, tamsulosin has been frequently used in patients with renal stones to possibly help with pain and promote more rapid stone passage. A recent randomized, controlled trial with 512 patients, authored by Andrew Meltzer, MD, and his colleagues, showed no improvement in stone passage rate in patients taking tamsulosin, compared with the rate seen with placebo.6

Previously published meta-analyses of multiple studies have shown a benefit to the use of alpha-blockers. Thijs Campschroer and colleagues included 67 studies that altogether included 10,509 participants.7 They found that the use of alpha-blockers led to possibly shorter stone expulsion times (3.4 days), less NSAID use, and fewer hospitalizations, with the evidence graded as low to moderate quality. Stone size seems to matter because use of alpha-blockers does not seem to make a difference for stones larger than 5 mm.

I think IV ketorolac would be the best of the options presented here for this patient. If a patient can safely take NSAIDs, those are probably the best option. There does not appear to be any reason to bolus hydrate patients with acute renal colic.

Dr. Paauw is a professor of medicine in the division of general internal medicine at the University of Washington, Seattle, and he serves as third-year medical student clerkship director at the University of Washington. Contact Dr. Paauw at dpaauw@uw.edu.

References

1. J Endourol. 2006 Oct;20(10):713-6.

2. Scand J Urol Nephrol. 1983;17(2):175-8.

3. Cochrane Database Syst Rev. 2012 Feb 15;(2):CD004926.

4. Ann Emerg Med. 1996 Aug;28(2):151-8.

5. BMJ. 2004 Jun 12;328(7453):1401.

6. JAMA Intern Med. 2018 Aug 1;178(8):1051-7.

7. Cochrane Database Syst Rev. 2018 Apr 5;4:CD008509.

Publications
Topics
Sections

A 40-year-old man presents with severe right flank pain for 1 hour. He has had this in the past when he passed a kidney stone. Urinalysis shows greater than 100 red blood cells per high power field (HPF). CT shows a 6-mm stone in the left ureter.

What do you recommend for therapy?

A. IV ketorolac and IV fluids.

B. IV morphine and IV fluids.

C. IV morphine.

D. IV ketorolac.

This is a common scenario, especially in emergency department settings and acute care clinics. Patients arrive in severe pain because of renal colic from kidney stones. Standard teaching that I received many years ago was that this patient should receive IV fluid to “help float the stone out” and narcotic pain medications to treat the severe pain the patient was in.

Paauw_Douglas_S_WA_web.jpg
Dr. Douglas S. Paauw

Is there good evidence that this is the best therapy?

There are scant data on the practice of IV fluid for treatment of renal stone passage. W. Patrick Springhart, MD, and his colleagues studied 43 patients who presented to the ED for treatment of renal colic.1 All patients had CT evaluation for stones and received intravenous analgesia. Twenty patients were randomized to receive 2 L of normal saline over 2 hours, and 23 patients received minimal IV saline (20 mL/hour). There were no differences between the two groups in pain scores, narcotic requirements, or stone passage rates.

In an older study, Tom-Harald Edna, PhD, and colleagues studied 60 patients with ureteral colic, randomizing them to receive either no fluid or 3 L of IV fluid over 6 hours.2 There was no significant difference in pain between treatment groups.

A Cochrane analysis in 2012 concluded that there was no reliable evidence to support the use of high-volume fluid therapy in the treatment of acute ureteral colic.3

Standard treatment of pain for renal colic has been to use narcotics. In a randomized, double-blind trial comparing ketorolac and meperidine, William Cordell, MD, and his colleagues found that pain relief was superior in ketorolac-treated patients. Seventy-five percent of ketorolac patients had a 50% reduction in pain scores versus only 23% of the patients who received meperidine (P less than .001).4

Anna Holdgate and Tamara Pollock reviewed 20 studies that evaluated NSAIDs and narcotics for acute renal colic. They concluded that patients treated with NSAIDs had greater pain relief with less vomiting than did patients treated with narcotics.5

In the past decade, tamsulosin has been frequently used in patients with renal stones to possibly help with pain and promote more rapid stone passage. A recent randomized, controlled trial with 512 patients, authored by Andrew Meltzer, MD, and his colleagues, showed no improvement in stone passage rate in patients taking tamsulosin, compared with the rate seen with placebo.6

Previously published meta-analyses of multiple studies have shown a benefit to the use of alpha-blockers. Thijs Campschroer and colleagues included 67 studies that altogether included 10,509 participants.7 They found that the use of alpha-blockers led to possibly shorter stone expulsion times (3.4 days), less NSAID use, and fewer hospitalizations, with the evidence graded as low to moderate quality. Stone size seems to matter because use of alpha-blockers does not seem to make a difference for stones larger than 5 mm.

I think IV ketorolac would be the best of the options presented here for this patient. If a patient can safely take NSAIDs, those are probably the best option. There does not appear to be any reason to bolus hydrate patients with acute renal colic.

Dr. Paauw is a professor of medicine in the division of general internal medicine at the University of Washington, Seattle, and he serves as third-year medical student clerkship director at the University of Washington. Contact Dr. Paauw at dpaauw@uw.edu.

References

1. J Endourol. 2006 Oct;20(10):713-6.

2. Scand J Urol Nephrol. 1983;17(2):175-8.

3. Cochrane Database Syst Rev. 2012 Feb 15;(2):CD004926.

4. Ann Emerg Med. 1996 Aug;28(2):151-8.

5. BMJ. 2004 Jun 12;328(7453):1401.

6. JAMA Intern Med. 2018 Aug 1;178(8):1051-7.

7. Cochrane Database Syst Rev. 2018 Apr 5;4:CD008509.

A 40-year-old man presents with severe right flank pain for 1 hour. He has had this in the past when he passed a kidney stone. Urinalysis shows greater than 100 red blood cells per high power field (HPF). CT shows a 6-mm stone in the left ureter.

What do you recommend for therapy?

A. IV ketorolac and IV fluids.

B. IV morphine and IV fluids.

C. IV morphine.

D. IV ketorolac.

This is a common scenario, especially in emergency department settings and acute care clinics. Patients arrive in severe pain because of renal colic from kidney stones. Standard teaching that I received many years ago was that this patient should receive IV fluid to “help float the stone out” and narcotic pain medications to treat the severe pain the patient was in.

Paauw_Douglas_S_WA_web.jpg
Dr. Douglas S. Paauw

Is there good evidence that this is the best therapy?

There are scant data on the practice of IV fluid for treatment of renal stone passage. W. Patrick Springhart, MD, and his colleagues studied 43 patients who presented to the ED for treatment of renal colic.1 All patients had CT evaluation for stones and received intravenous analgesia. Twenty patients were randomized to receive 2 L of normal saline over 2 hours, and 23 patients received minimal IV saline (20 mL/hour). There were no differences between the two groups in pain scores, narcotic requirements, or stone passage rates.

In an older study, Tom-Harald Edna, PhD, and colleagues studied 60 patients with ureteral colic, randomizing them to receive either no fluid or 3 L of IV fluid over 6 hours.2 There was no significant difference in pain between treatment groups.

A Cochrane analysis in 2012 concluded that there was no reliable evidence to support the use of high-volume fluid therapy in the treatment of acute ureteral colic.3

Standard treatment of pain for renal colic has been to use narcotics. In a randomized, double-blind trial comparing ketorolac and meperidine, William Cordell, MD, and his colleagues found that pain relief was superior in ketorolac-treated patients. Seventy-five percent of ketorolac patients had a 50% reduction in pain scores versus only 23% of the patients who received meperidine (P less than .001).4

Anna Holdgate and Tamara Pollock reviewed 20 studies that evaluated NSAIDs and narcotics for acute renal colic. They concluded that patients treated with NSAIDs had greater pain relief with less vomiting than did patients treated with narcotics.5

In the past decade, tamsulosin has been frequently used in patients with renal stones to possibly help with pain and promote more rapid stone passage. A recent randomized, controlled trial with 512 patients, authored by Andrew Meltzer, MD, and his colleagues, showed no improvement in stone passage rate in patients taking tamsulosin, compared with the rate seen with placebo.6

Previously published meta-analyses of multiple studies have shown a benefit to the use of alpha-blockers. Thijs Campschroer and colleagues included 67 studies that altogether included 10,509 participants.7 They found that the use of alpha-blockers led to possibly shorter stone expulsion times (3.4 days), less NSAID use, and fewer hospitalizations, with the evidence graded as low to moderate quality. Stone size seems to matter because use of alpha-blockers does not seem to make a difference for stones larger than 5 mm.

I think IV ketorolac would be the best of the options presented here for this patient. If a patient can safely take NSAIDs, those are probably the best option. There does not appear to be any reason to bolus hydrate patients with acute renal colic.

Dr. Paauw is a professor of medicine in the division of general internal medicine at the University of Washington, Seattle, and he serves as third-year medical student clerkship director at the University of Washington. Contact Dr. Paauw at dpaauw@uw.edu.

References

1. J Endourol. 2006 Oct;20(10):713-6.

2. Scand J Urol Nephrol. 1983;17(2):175-8.

3. Cochrane Database Syst Rev. 2012 Feb 15;(2):CD004926.

4. Ann Emerg Med. 1996 Aug;28(2):151-8.

5. BMJ. 2004 Jun 12;328(7453):1401.

6. JAMA Intern Med. 2018 Aug 1;178(8):1051-7.

7. Cochrane Database Syst Rev. 2018 Apr 5;4:CD008509.

Publications
Publications
Topics
Article Type
Sections
Disallow All Ads
Content Gating
No Gating (article Unlocked/Free)
Alternative CME
Disqus Comments
Default
Use ProPublica

Myth of the Month: MONA for acute treatment of chest pain?

Article Type
Changed
Fri, 01/18/2019 - 17:13

 

A 66-year-old man presents with substernal chest pressure and dyspnea that has been present for 45 minutes. He has nausea. Vital signs: blood pressure, 110/60; pulse, 100; oxygen saturation, 92%. Neck: elevated jugular venous pressure. Chest: clear. Cardiac: normal S1 S2, no murmurs. ECG: ST elevation in 2, 3, and aVF leads.

Which of these treatments do you recommend?

A. Morphine, oxygen, nitroglycerin, and aspirin (ASA).

B. Oxygen, morphine, ASA.

C. ASA.

Tubbesing_Greta_WA_web.TIF
Greta Tubbesing, MD
For many years, a standard treatment of providing morphine, oxygen, nitroglycerin, and aspirin (MONA) was the standard initial treatment approach for all patients presenting with chest pain due to suspected myocardial ischemia.

In this patient, I think the correct approach would be to just give aspirin. Nitroglycerin would be problematic, as it appears that this patient might be having a right ventricular infarct, and lowering right-sided filling pressures with nitroglycerin may lead to severe hypotension.

There is controversy over the safety of routine morphine use for patients with chest pain.

Trip J. Meine, MD, and colleagues found that use of morphine either alone or in combination with nitroglycerin for patients presenting with non–ST-elevation acute coronary syndrome (NSTE-ACS) was associated with higher mortality.1 Cian P. McCarthy, MD, and colleagues found the same results, with morphine use associated with larger infarct size, a longer hospital stay, and a trend toward increased mortality in invasively managed NSTE-ACS patients.2 Suzanne de Waha and colleagues found that morphine use in patients with ST-segment elevation MIs had larger infarct size and less reperfusion success, as measured by cardiac MRI.3

Not all recent studies show a detrimental effect of morphine. Etienne Puymirat et al. reviewed in-hospital complications (death, nonfatal re-MI, stroke, stent thrombosis, and bleeding) and 1-year survival according to prehospital morphine use in 2,438 ST-elevation MI (STEMI) patients from the French Registry of Acute ST-elevation and non–ST-elevation Myocardial Infarction (FAST-MI).4 They found no increase in in-hospital complications or 1-year mortality.

The practice of using supplemental oxygen to treat all patients with MI became standard nearly a century ago, after oxygen was found in 1900 to relieve angina, and led to clinical improvement in four MI patients in a 1930 case series.5,6

It was not studied in a controlled trial until 1976, when J.M. Rawles, MD, and colleagues randomized 157 patients with MI to 24 hours of oxygen at 8 L/min or to ambient air. They found no difference in mortality between the groups, but they did find a higher burden of MI in the intervention arm receiving supplemental oxygen, as measured by mean serum aspartate aminotransferase levels.7

The topic was not addressed again in a significant randomized trial until this century. Most notably, two recent studies again demonstrated no benefit of supplemental oxygen in normoxemic patients with MI.

In the AVOID trial in 2015, Dion Stub, MD, PhD, and colleagues randomized 441 patients with STEMI to oxygen at 8 L/min – from diagnosis in an ambulance until after cardiac catheterization – or to ambient air. They found no difference in death at 6 months, but did find an increased rate of in-hospital recurrent MIs, with 0.9% of the control group and 5.5% of the oxygen intervention arm suffering recurrence (P = .006).8 They also showed a larger area of myocardial infarct in the oxygen group, as measured by peak creatine kinase levels and cardiac MRI at 6 months.

Proposed mechanisms of increased myocardial injury from hyperoxia include increased coronary vascular resistance resulting in decreased myocardial perfusion, and increased reperfusion injury from formation of free radicals.9

Paauw_Douglas_S_WA_web.jpg
Dr. Douglas S. Paauw
This year, a large randomized trial of 6,629 patients across 35 Swedish hospitals was published by Robin Hofmann, MD, and colleagues.10 The DETO2X-AMI study compared 6 L/min of oxygen delivered for an average of 11.6 hours to ambient air in normoxemic patients with suspected MI (76% with ultimately confirmed MI). They found no difference in death at 30 days or 1 year. While this finding reinforced the lack of benefit of supplemental oxygen shown in the AVOID trial, the findings by Dr. Stub and colleagues of increased tissue damage were not borne out: Both groups showed similar troponin levels.

Where does all this leave us in the treatment of suspected MI?

Morphine should only be used when the patient has pain, and is probably best reserved for severe pain, as the safety of its use is not clear. While hypoxemia is a common consequence of MI – and may correlate with worse outcomes – treatment with supplemental oxygen in the absence of hypoxemia is not supported by current evidence, and may carry risk of harm. Nitroglycerin should be avoided in patients with right ventricular infarcts, and in patients who present with hypotension.
 

 

 

Dr. Tubbesing is a senior resident in medicine at the University of Washington, Seattle. Dr. Paauw is professor of medicine in the division of general internal medicine at the University of Washington, Seattle, and he serves as third-year medical student clerkship director at the University of Washington. Contact Dr. Paauw at dpaauw@uw.edu.

References

1. Am Heart J. 2005 Jun;149(6):1043-9.

2. J Interv Cardiol. 2017 Nov 22. doi: 10.1111/joic.12464.

3. Clin Res Cardiol. 2015 Sep;104(9):727-34.

4. Eur Heart J. 2016 Apr 1;37(13):1063-71.

5. BMJ. 1900 Dec 1;2(2083):1568.

6. JAMA. 1930 May 3;94(18):1363-5.

7. Br Med J. 1976 May 8;1(6018):1121-3.

8. Circulation. 2015 Jun 16;131(24):2143-50.

9. Cochrane Database Syst Rev. 2016 Dec 19;12:CD007160.

10. N Engl J Med. 2017 Sep 28;377(13):1240-9.

Publications
Topics
Sections

 

A 66-year-old man presents with substernal chest pressure and dyspnea that has been present for 45 minutes. He has nausea. Vital signs: blood pressure, 110/60; pulse, 100; oxygen saturation, 92%. Neck: elevated jugular venous pressure. Chest: clear. Cardiac: normal S1 S2, no murmurs. ECG: ST elevation in 2, 3, and aVF leads.

Which of these treatments do you recommend?

A. Morphine, oxygen, nitroglycerin, and aspirin (ASA).

B. Oxygen, morphine, ASA.

C. ASA.

Tubbesing_Greta_WA_web.TIF
Greta Tubbesing, MD
For many years, a standard treatment of providing morphine, oxygen, nitroglycerin, and aspirin (MONA) was the standard initial treatment approach for all patients presenting with chest pain due to suspected myocardial ischemia.

In this patient, I think the correct approach would be to just give aspirin. Nitroglycerin would be problematic, as it appears that this patient might be having a right ventricular infarct, and lowering right-sided filling pressures with nitroglycerin may lead to severe hypotension.

There is controversy over the safety of routine morphine use for patients with chest pain.

Trip J. Meine, MD, and colleagues found that use of morphine either alone or in combination with nitroglycerin for patients presenting with non–ST-elevation acute coronary syndrome (NSTE-ACS) was associated with higher mortality.1 Cian P. McCarthy, MD, and colleagues found the same results, with morphine use associated with larger infarct size, a longer hospital stay, and a trend toward increased mortality in invasively managed NSTE-ACS patients.2 Suzanne de Waha and colleagues found that morphine use in patients with ST-segment elevation MIs had larger infarct size and less reperfusion success, as measured by cardiac MRI.3

Not all recent studies show a detrimental effect of morphine. Etienne Puymirat et al. reviewed in-hospital complications (death, nonfatal re-MI, stroke, stent thrombosis, and bleeding) and 1-year survival according to prehospital morphine use in 2,438 ST-elevation MI (STEMI) patients from the French Registry of Acute ST-elevation and non–ST-elevation Myocardial Infarction (FAST-MI).4 They found no increase in in-hospital complications or 1-year mortality.

The practice of using supplemental oxygen to treat all patients with MI became standard nearly a century ago, after oxygen was found in 1900 to relieve angina, and led to clinical improvement in four MI patients in a 1930 case series.5,6

It was not studied in a controlled trial until 1976, when J.M. Rawles, MD, and colleagues randomized 157 patients with MI to 24 hours of oxygen at 8 L/min or to ambient air. They found no difference in mortality between the groups, but they did find a higher burden of MI in the intervention arm receiving supplemental oxygen, as measured by mean serum aspartate aminotransferase levels.7

The topic was not addressed again in a significant randomized trial until this century. Most notably, two recent studies again demonstrated no benefit of supplemental oxygen in normoxemic patients with MI.

In the AVOID trial in 2015, Dion Stub, MD, PhD, and colleagues randomized 441 patients with STEMI to oxygen at 8 L/min – from diagnosis in an ambulance until after cardiac catheterization – or to ambient air. They found no difference in death at 6 months, but did find an increased rate of in-hospital recurrent MIs, with 0.9% of the control group and 5.5% of the oxygen intervention arm suffering recurrence (P = .006).8 They also showed a larger area of myocardial infarct in the oxygen group, as measured by peak creatine kinase levels and cardiac MRI at 6 months.

Proposed mechanisms of increased myocardial injury from hyperoxia include increased coronary vascular resistance resulting in decreased myocardial perfusion, and increased reperfusion injury from formation of free radicals.9

Paauw_Douglas_S_WA_web.jpg
Dr. Douglas S. Paauw
This year, a large randomized trial of 6,629 patients across 35 Swedish hospitals was published by Robin Hofmann, MD, and colleagues.10 The DETO2X-AMI study compared 6 L/min of oxygen delivered for an average of 11.6 hours to ambient air in normoxemic patients with suspected MI (76% with ultimately confirmed MI). They found no difference in death at 30 days or 1 year. While this finding reinforced the lack of benefit of supplemental oxygen shown in the AVOID trial, the findings by Dr. Stub and colleagues of increased tissue damage were not borne out: Both groups showed similar troponin levels.

Where does all this leave us in the treatment of suspected MI?

Morphine should only be used when the patient has pain, and is probably best reserved for severe pain, as the safety of its use is not clear. While hypoxemia is a common consequence of MI – and may correlate with worse outcomes – treatment with supplemental oxygen in the absence of hypoxemia is not supported by current evidence, and may carry risk of harm. Nitroglycerin should be avoided in patients with right ventricular infarcts, and in patients who present with hypotension.
 

 

 

Dr. Tubbesing is a senior resident in medicine at the University of Washington, Seattle. Dr. Paauw is professor of medicine in the division of general internal medicine at the University of Washington, Seattle, and he serves as third-year medical student clerkship director at the University of Washington. Contact Dr. Paauw at dpaauw@uw.edu.

References

1. Am Heart J. 2005 Jun;149(6):1043-9.

2. J Interv Cardiol. 2017 Nov 22. doi: 10.1111/joic.12464.

3. Clin Res Cardiol. 2015 Sep;104(9):727-34.

4. Eur Heart J. 2016 Apr 1;37(13):1063-71.

5. BMJ. 1900 Dec 1;2(2083):1568.

6. JAMA. 1930 May 3;94(18):1363-5.

7. Br Med J. 1976 May 8;1(6018):1121-3.

8. Circulation. 2015 Jun 16;131(24):2143-50.

9. Cochrane Database Syst Rev. 2016 Dec 19;12:CD007160.

10. N Engl J Med. 2017 Sep 28;377(13):1240-9.

 

A 66-year-old man presents with substernal chest pressure and dyspnea that has been present for 45 minutes. He has nausea. Vital signs: blood pressure, 110/60; pulse, 100; oxygen saturation, 92%. Neck: elevated jugular venous pressure. Chest: clear. Cardiac: normal S1 S2, no murmurs. ECG: ST elevation in 2, 3, and aVF leads.

Which of these treatments do you recommend?

A. Morphine, oxygen, nitroglycerin, and aspirin (ASA).

B. Oxygen, morphine, ASA.

C. ASA.

Tubbesing_Greta_WA_web.TIF
Greta Tubbesing, MD
For many years, a standard treatment of providing morphine, oxygen, nitroglycerin, and aspirin (MONA) was the standard initial treatment approach for all patients presenting with chest pain due to suspected myocardial ischemia.

In this patient, I think the correct approach would be to just give aspirin. Nitroglycerin would be problematic, as it appears that this patient might be having a right ventricular infarct, and lowering right-sided filling pressures with nitroglycerin may lead to severe hypotension.

There is controversy over the safety of routine morphine use for patients with chest pain.

Trip J. Meine, MD, and colleagues found that use of morphine either alone or in combination with nitroglycerin for patients presenting with non–ST-elevation acute coronary syndrome (NSTE-ACS) was associated with higher mortality.1 Cian P. McCarthy, MD, and colleagues found the same results, with morphine use associated with larger infarct size, a longer hospital stay, and a trend toward increased mortality in invasively managed NSTE-ACS patients.2 Suzanne de Waha and colleagues found that morphine use in patients with ST-segment elevation MIs had larger infarct size and less reperfusion success, as measured by cardiac MRI.3

Not all recent studies show a detrimental effect of morphine. Etienne Puymirat et al. reviewed in-hospital complications (death, nonfatal re-MI, stroke, stent thrombosis, and bleeding) and 1-year survival according to prehospital morphine use in 2,438 ST-elevation MI (STEMI) patients from the French Registry of Acute ST-elevation and non–ST-elevation Myocardial Infarction (FAST-MI).4 They found no increase in in-hospital complications or 1-year mortality.

The practice of using supplemental oxygen to treat all patients with MI became standard nearly a century ago, after oxygen was found in 1900 to relieve angina, and led to clinical improvement in four MI patients in a 1930 case series.5,6

It was not studied in a controlled trial until 1976, when J.M. Rawles, MD, and colleagues randomized 157 patients with MI to 24 hours of oxygen at 8 L/min or to ambient air. They found no difference in mortality between the groups, but they did find a higher burden of MI in the intervention arm receiving supplemental oxygen, as measured by mean serum aspartate aminotransferase levels.7

The topic was not addressed again in a significant randomized trial until this century. Most notably, two recent studies again demonstrated no benefit of supplemental oxygen in normoxemic patients with MI.

In the AVOID trial in 2015, Dion Stub, MD, PhD, and colleagues randomized 441 patients with STEMI to oxygen at 8 L/min – from diagnosis in an ambulance until after cardiac catheterization – or to ambient air. They found no difference in death at 6 months, but did find an increased rate of in-hospital recurrent MIs, with 0.9% of the control group and 5.5% of the oxygen intervention arm suffering recurrence (P = .006).8 They also showed a larger area of myocardial infarct in the oxygen group, as measured by peak creatine kinase levels and cardiac MRI at 6 months.

Proposed mechanisms of increased myocardial injury from hyperoxia include increased coronary vascular resistance resulting in decreased myocardial perfusion, and increased reperfusion injury from formation of free radicals.9

Paauw_Douglas_S_WA_web.jpg
Dr. Douglas S. Paauw
This year, a large randomized trial of 6,629 patients across 35 Swedish hospitals was published by Robin Hofmann, MD, and colleagues.10 The DETO2X-AMI study compared 6 L/min of oxygen delivered for an average of 11.6 hours to ambient air in normoxemic patients with suspected MI (76% with ultimately confirmed MI). They found no difference in death at 30 days or 1 year. While this finding reinforced the lack of benefit of supplemental oxygen shown in the AVOID trial, the findings by Dr. Stub and colleagues of increased tissue damage were not borne out: Both groups showed similar troponin levels.

Where does all this leave us in the treatment of suspected MI?

Morphine should only be used when the patient has pain, and is probably best reserved for severe pain, as the safety of its use is not clear. While hypoxemia is a common consequence of MI – and may correlate with worse outcomes – treatment with supplemental oxygen in the absence of hypoxemia is not supported by current evidence, and may carry risk of harm. Nitroglycerin should be avoided in patients with right ventricular infarcts, and in patients who present with hypotension.
 

 

 

Dr. Tubbesing is a senior resident in medicine at the University of Washington, Seattle. Dr. Paauw is professor of medicine in the division of general internal medicine at the University of Washington, Seattle, and he serves as third-year medical student clerkship director at the University of Washington. Contact Dr. Paauw at dpaauw@uw.edu.

References

1. Am Heart J. 2005 Jun;149(6):1043-9.

2. J Interv Cardiol. 2017 Nov 22. doi: 10.1111/joic.12464.

3. Clin Res Cardiol. 2015 Sep;104(9):727-34.

4. Eur Heart J. 2016 Apr 1;37(13):1063-71.

5. BMJ. 1900 Dec 1;2(2083):1568.

6. JAMA. 1930 May 3;94(18):1363-5.

7. Br Med J. 1976 May 8;1(6018):1121-3.

8. Circulation. 2015 Jun 16;131(24):2143-50.

9. Cochrane Database Syst Rev. 2016 Dec 19;12:CD007160.

10. N Engl J Med. 2017 Sep 28;377(13):1240-9.

Publications
Publications
Topics
Article Type
Sections
Disallow All Ads
Content Gating
No Gating (article Unlocked/Free)
Alternative CME
Disqus Comments
Default

Double the dose of antihypertensive meds?

Article Type
Changed
Fri, 01/18/2019 - 16:46


A 50-year-old man returns for follow-up of hypertension. He is currently taking 20 mg of lisinopril. His blood pressure readings over the past month are 150/96, 155/98, 160/94, and 162/96. His renal function is normal, and he has been taking his lisinopril regularly.

What do you recommend?

A. Increase his lisinopril to 20 mg twice a day.

B. Switch to valsartan.

C. Add amlodipine.

Paauw_Douglas_S_WA_web.jpg
Dr. Douglas S. Paauw
For many years, we have been taught to always maximize the dose of one medication before adding another medication. This has historically been the case for hypertension management.1 The thought process has been that, if you add an additional medication, you add more potential side effects and added cost.

But is there much benefit in doubling the dose of antihypertensive medications?

H.J. Gomez and colleagues studied the dose response of lisinopril in essential hypertension.2 Patients received very-low-dose (1.25 mg or 5 mg), moderate-dose (20 mg), or high-dose (80 mg) lisinopril. The difference in blood pressure reduction between 20 mg and 80 mg was modest (5 mm/3 mm less in those receiving 80 mg, compared with 20 mg). There was no clinical effect at 1.25 mg of lisinopril, but a relatively flat dose response above 20 mg.

A similar finding was reported by J.R. Benz and colleagues in regard to escalating doses of valsartan.3 The study looked at blood pressure in response to valsartan at doses of 80 mg and 160 mg, and in combination with hydrochlorothiazide. The difference in blood pressure between valsartan 160 mg and 80 mg was 3 mm/0.8 mm. The difference in blood pressure between patients taking 80 mg of valsartan and 25 mg hydrochlorothiazide, compared with those taking 80 mg of valsartan, was 12/6.

In a meta-analysis of 354 randomized trials of fixed-dose blood pressure medications, M.R. Law and colleagues found that cutting the doses in half only reduced effectiveness of lowering BP by 20%.4 The average reduction in systolic BP was 9.1 mm Hg, and reduction in diastolic BP was 5.5mm Hg – which only was reduced to 7.1 mm Hg/4.4 mm Hg when the doses of medications were cut in half. Side effects attributed to beta-blockers, calcium channel blockers, and diuretics were very dose related, whereas the side effects attributed to ACE inhibitors were not.

In another meta-analysis comparing monotherapy vs. combination therapy for lowering blood pressure, adding another drug lowered blood pressure fivefold more than doubling the dose of the initial antihypertensive drug.5

I think the right answer in this case would be to add amlodipine instead of doubling the dose of lisinopril or switching to valsartan as a single agent. The data are striking on how little effect there is in increasing antihypertensive medication doses. Adding another antihypertensive medication should be the standard practice when the first medication started does not achieve the desired goal.
 

Dr. Paauw is professor of medicine in the division of general internal medicine at the University of Washington, Seattle, and he serves as third-year medical student clerkship director at the University of Washington. Contact Dr. Paauw at dpaauw@uw.edu.

References

1. JAMA. 2003 May 21;289(19):2560-72.

2. Br J Clin Pharm. 1989;28:415-20.

3. J Hum Hypertens. 1998 Dec;12(12):861-6.

4. BMJ. 2003 Jun 28;326(7404):1427.

5. Am J Med. 2009 Mar;122(3):290-300.

Publications
Topics
Sections


A 50-year-old man returns for follow-up of hypertension. He is currently taking 20 mg of lisinopril. His blood pressure readings over the past month are 150/96, 155/98, 160/94, and 162/96. His renal function is normal, and he has been taking his lisinopril regularly.

What do you recommend?

A. Increase his lisinopril to 20 mg twice a day.

B. Switch to valsartan.

C. Add amlodipine.

Paauw_Douglas_S_WA_web.jpg
Dr. Douglas S. Paauw
For many years, we have been taught to always maximize the dose of one medication before adding another medication. This has historically been the case for hypertension management.1 The thought process has been that, if you add an additional medication, you add more potential side effects and added cost.

But is there much benefit in doubling the dose of antihypertensive medications?

H.J. Gomez and colleagues studied the dose response of lisinopril in essential hypertension.2 Patients received very-low-dose (1.25 mg or 5 mg), moderate-dose (20 mg), or high-dose (80 mg) lisinopril. The difference in blood pressure reduction between 20 mg and 80 mg was modest (5 mm/3 mm less in those receiving 80 mg, compared with 20 mg). There was no clinical effect at 1.25 mg of lisinopril, but a relatively flat dose response above 20 mg.

A similar finding was reported by J.R. Benz and colleagues in regard to escalating doses of valsartan.3 The study looked at blood pressure in response to valsartan at doses of 80 mg and 160 mg, and in combination with hydrochlorothiazide. The difference in blood pressure between valsartan 160 mg and 80 mg was 3 mm/0.8 mm. The difference in blood pressure between patients taking 80 mg of valsartan and 25 mg hydrochlorothiazide, compared with those taking 80 mg of valsartan, was 12/6.

In a meta-analysis of 354 randomized trials of fixed-dose blood pressure medications, M.R. Law and colleagues found that cutting the doses in half only reduced effectiveness of lowering BP by 20%.4 The average reduction in systolic BP was 9.1 mm Hg, and reduction in diastolic BP was 5.5mm Hg – which only was reduced to 7.1 mm Hg/4.4 mm Hg when the doses of medications were cut in half. Side effects attributed to beta-blockers, calcium channel blockers, and diuretics were very dose related, whereas the side effects attributed to ACE inhibitors were not.

In another meta-analysis comparing monotherapy vs. combination therapy for lowering blood pressure, adding another drug lowered blood pressure fivefold more than doubling the dose of the initial antihypertensive drug.5

I think the right answer in this case would be to add amlodipine instead of doubling the dose of lisinopril or switching to valsartan as a single agent. The data are striking on how little effect there is in increasing antihypertensive medication doses. Adding another antihypertensive medication should be the standard practice when the first medication started does not achieve the desired goal.
 

Dr. Paauw is professor of medicine in the division of general internal medicine at the University of Washington, Seattle, and he serves as third-year medical student clerkship director at the University of Washington. Contact Dr. Paauw at dpaauw@uw.edu.

References

1. JAMA. 2003 May 21;289(19):2560-72.

2. Br J Clin Pharm. 1989;28:415-20.

3. J Hum Hypertens. 1998 Dec;12(12):861-6.

4. BMJ. 2003 Jun 28;326(7404):1427.

5. Am J Med. 2009 Mar;122(3):290-300.


A 50-year-old man returns for follow-up of hypertension. He is currently taking 20 mg of lisinopril. His blood pressure readings over the past month are 150/96, 155/98, 160/94, and 162/96. His renal function is normal, and he has been taking his lisinopril regularly.

What do you recommend?

A. Increase his lisinopril to 20 mg twice a day.

B. Switch to valsartan.

C. Add amlodipine.

Paauw_Douglas_S_WA_web.jpg
Dr. Douglas S. Paauw
For many years, we have been taught to always maximize the dose of one medication before adding another medication. This has historically been the case for hypertension management.1 The thought process has been that, if you add an additional medication, you add more potential side effects and added cost.

But is there much benefit in doubling the dose of antihypertensive medications?

H.J. Gomez and colleagues studied the dose response of lisinopril in essential hypertension.2 Patients received very-low-dose (1.25 mg or 5 mg), moderate-dose (20 mg), or high-dose (80 mg) lisinopril. The difference in blood pressure reduction between 20 mg and 80 mg was modest (5 mm/3 mm less in those receiving 80 mg, compared with 20 mg). There was no clinical effect at 1.25 mg of lisinopril, but a relatively flat dose response above 20 mg.

A similar finding was reported by J.R. Benz and colleagues in regard to escalating doses of valsartan.3 The study looked at blood pressure in response to valsartan at doses of 80 mg and 160 mg, and in combination with hydrochlorothiazide. The difference in blood pressure between valsartan 160 mg and 80 mg was 3 mm/0.8 mm. The difference in blood pressure between patients taking 80 mg of valsartan and 25 mg hydrochlorothiazide, compared with those taking 80 mg of valsartan, was 12/6.

In a meta-analysis of 354 randomized trials of fixed-dose blood pressure medications, M.R. Law and colleagues found that cutting the doses in half only reduced effectiveness of lowering BP by 20%.4 The average reduction in systolic BP was 9.1 mm Hg, and reduction in diastolic BP was 5.5mm Hg – which only was reduced to 7.1 mm Hg/4.4 mm Hg when the doses of medications were cut in half. Side effects attributed to beta-blockers, calcium channel blockers, and diuretics were very dose related, whereas the side effects attributed to ACE inhibitors were not.

In another meta-analysis comparing monotherapy vs. combination therapy for lowering blood pressure, adding another drug lowered blood pressure fivefold more than doubling the dose of the initial antihypertensive drug.5

I think the right answer in this case would be to add amlodipine instead of doubling the dose of lisinopril or switching to valsartan as a single agent. The data are striking on how little effect there is in increasing antihypertensive medication doses. Adding another antihypertensive medication should be the standard practice when the first medication started does not achieve the desired goal.
 

Dr. Paauw is professor of medicine in the division of general internal medicine at the University of Washington, Seattle, and he serves as third-year medical student clerkship director at the University of Washington. Contact Dr. Paauw at dpaauw@uw.edu.

References

1. JAMA. 2003 May 21;289(19):2560-72.

2. Br J Clin Pharm. 1989;28:415-20.

3. J Hum Hypertens. 1998 Dec;12(12):861-6.

4. BMJ. 2003 Jun 28;326(7404):1427.

5. Am J Med. 2009 Mar;122(3):290-300.

Publications
Publications
Topics
Article Type
Sections
Disallow All Ads
Content Gating
No Gating (article Unlocked/Free)
Alternative CME

Antibiotic prophylaxis for artificial joints

Article Type
Changed
Wed, 03/13/2019 - 15:02


A 66-year-old woman 3 years status post hip replacement is seen for dental work. The dentist contacts the clinic for an antibiotic prescription. The patient has a penicillin allergy (rash). What do you recommend?

A. Clindamycin one dose before dental work.

B. Amoxicillin one dose before dental work.

C. Amoxicillin one dose before, one dose 4 hours after dental work.

D. Clindamycin one dose before dental work, one dose 4 hours after dental work.

E. No antibiotics.

Many patients with prosthetic joints will request antibiotics to take prior to dental procedures. Sometimes this request comes from the dental office.

When I ask patients why they feel they need antibiotics, they often reply that they were told by their orthopedic surgeons or their dentist that they would need to take antibiotics before dental procedures.

In an era when Clostridium difficile infection is a common and dangerous complication in the elderly, avoidance of unnecessary antibiotics is critical. In the United States, it is estimated that there are 240,000 patients infected with C. difficile annually, with 24,000 deaths at a cost of $6 billion.1

Is there compelling evidence to justify giving antibiotic prophylaxis for dental procedures to patients with prosthetic joints?

Paauw_Douglas_S_WA_web.jpg
Dr. Douglas S. Paauw
The majority of prosthetic joint infections are due to Staphylococcus aureus, whereas the majority of bacteremias from dental procedures are due to streptococcus.1,2 Bacteremias following simple everyday activities such as tooth brushing and chewing occur.3

This information has called into question the wisdom of giving antibiotic prophylaxis for dental procedures when the same patients have transient bacteremias as a regular part of day-to-day life, and mouth organisms were infrequent causes of prosthetic joint infections.

The American Academy of Orthopaedic Surgeons (AAOS) and the American Dental Association (ADA) released an advisory statement 20 years ago on antibiotic prophylaxis for patients with dental replacements, which concluded: “Antibiotic prophylaxis is not indicated for dental patients with pins, plates, and screws, nor is it routinely indicated for most dental patients with total joint replacements.”4

In 2003, the AAOS and the ADA released updated guidelines that stated: “Presently, no scientific evidence supports the position that antibiotic prophylaxis to prevent hematogenous infections is required prior to dental treatment in patients with total joint prostheses. The risk/benefit and cost/effectiveness ratios fail to justify the administration of routine antibiotics.”5

Great confusion arose in 2009 when the AAOS published a position paper on its website that reversed this position.6 Interestingly, the statement was done by the AAOS alone, and not done in conjunction with the ADA.

In this position paper, the AAOS recommended that health care providers consider antibiotic prophylaxis prior to invasive procedures on all patients who had prosthetic joints, regardless of how long those joints have been in place. This major change in recommendations was not based on any new evidence that had been reviewed since the 2003 guidelines.

There are two studies that address outcome of patients with prosthetic joints who have and have not received prophylactic antibiotics.

Elie Berbari, MD, and colleagues reported on the results of a prospective case-control study comparing patients with prosthetic joints hospitalized with hip or knee infections with patients who had prosthetic joints hospitalized at the same time who did not have hip or knee infections.7

There was no increased risk of prosthetic hip or knee infection for patients undergoing a dental procedure who were not receiving antibiotic prophylaxis (odds ratio, 0.8; 95% confidence interval, 0.4-1.6), compared with the risk for patients not undergoing a dental procedure (OR, 0.6; 95% CI, 0.4-1.1). Antibiotic prophylaxis in patients undergoing high and low risk dental procedures did not decrease the risk of prosthetic joint infections.

In 2012, the AAOS and the ADA published updated guidelines with the following summary recommendation: “The practitioner might consider discontinuing the practice of routinely prescribing prophylactic antibiotics for patients with hip and knee prosthetic joint implants undergoing dental procedures.”8 They referenced the Berbari study as the best available evidence.

Feng-Chen Kao, MD, and colleagues published a study this year with a design very similar to the Berbari study, with similar results.9 All Taiwanese residents who had received hip or knee replacements over a 12-year period were screened. Those who had received dental procedures were matched with individuals who had not had dental procedures. The dental procedure group was subdivided into a group that received antibiotics and one that didn’t.

There was no difference in infection rates between the group that had received dental procedures and the group that did not, and no difference in infection rates between those who received prophylactic antibiotics and those who didn’t.

I think this myth can be put to rest. There is no evidence to give patients with joint prostheses prophylactic antibiotics before dental procedures.


 

 

 

References

1. Steckelberg J.M., Osmon D.R. Prosthetic joint infections. In: Bisno A.L., Waldvogel F.A., eds. Infections associated with indwelling medical devices. Third ed., Washington, D.C.: American Society of Microbiology Press, 2000:173-209.

2. J Dent Res. 2004 Feb;83(2):170-4.

3. J Clin Periodontol. 2006 Feb;33(6):401-7.

4. J Am Dent Assoc. 1997 Jul;128(7):1004-8.

5. J Am Dent Assoc. 2003 Jul;134(7):895-9.

6. Spec Care Dentist. 2009 Nov-Dec;29(6):229-31.

7. Clin Infect Dis. 2010 Jan 1;50(1):8-16.

8. J Dent (Shiraz). 2013 Mar;14(1):49-52.

9. Infect Control Hosp Epidemiol. 2017 Feb;38(2):154-61.

 

Dr. Paauw is professor of medicine in the division of general internal medicine at the University of Washington, Seattle, and he serves as third-year medical student clerkship director at the University of Washington. Contact Dr. Paauw at dpaauw@uw.edu.

Publications
Topics
Sections


A 66-year-old woman 3 years status post hip replacement is seen for dental work. The dentist contacts the clinic for an antibiotic prescription. The patient has a penicillin allergy (rash). What do you recommend?

A. Clindamycin one dose before dental work.

B. Amoxicillin one dose before dental work.

C. Amoxicillin one dose before, one dose 4 hours after dental work.

D. Clindamycin one dose before dental work, one dose 4 hours after dental work.

E. No antibiotics.

Many patients with prosthetic joints will request antibiotics to take prior to dental procedures. Sometimes this request comes from the dental office.

When I ask patients why they feel they need antibiotics, they often reply that they were told by their orthopedic surgeons or their dentist that they would need to take antibiotics before dental procedures.

In an era when Clostridium difficile infection is a common and dangerous complication in the elderly, avoidance of unnecessary antibiotics is critical. In the United States, it is estimated that there are 240,000 patients infected with C. difficile annually, with 24,000 deaths at a cost of $6 billion.1

Is there compelling evidence to justify giving antibiotic prophylaxis for dental procedures to patients with prosthetic joints?

Paauw_Douglas_S_WA_web.jpg
Dr. Douglas S. Paauw
The majority of prosthetic joint infections are due to Staphylococcus aureus, whereas the majority of bacteremias from dental procedures are due to streptococcus.1,2 Bacteremias following simple everyday activities such as tooth brushing and chewing occur.3

This information has called into question the wisdom of giving antibiotic prophylaxis for dental procedures when the same patients have transient bacteremias as a regular part of day-to-day life, and mouth organisms were infrequent causes of prosthetic joint infections.

The American Academy of Orthopaedic Surgeons (AAOS) and the American Dental Association (ADA) released an advisory statement 20 years ago on antibiotic prophylaxis for patients with dental replacements, which concluded: “Antibiotic prophylaxis is not indicated for dental patients with pins, plates, and screws, nor is it routinely indicated for most dental patients with total joint replacements.”4

In 2003, the AAOS and the ADA released updated guidelines that stated: “Presently, no scientific evidence supports the position that antibiotic prophylaxis to prevent hematogenous infections is required prior to dental treatment in patients with total joint prostheses. The risk/benefit and cost/effectiveness ratios fail to justify the administration of routine antibiotics.”5

Great confusion arose in 2009 when the AAOS published a position paper on its website that reversed this position.6 Interestingly, the statement was done by the AAOS alone, and not done in conjunction with the ADA.

In this position paper, the AAOS recommended that health care providers consider antibiotic prophylaxis prior to invasive procedures on all patients who had prosthetic joints, regardless of how long those joints have been in place. This major change in recommendations was not based on any new evidence that had been reviewed since the 2003 guidelines.

There are two studies that address outcome of patients with prosthetic joints who have and have not received prophylactic antibiotics.

Elie Berbari, MD, and colleagues reported on the results of a prospective case-control study comparing patients with prosthetic joints hospitalized with hip or knee infections with patients who had prosthetic joints hospitalized at the same time who did not have hip or knee infections.7

There was no increased risk of prosthetic hip or knee infection for patients undergoing a dental procedure who were not receiving antibiotic prophylaxis (odds ratio, 0.8; 95% confidence interval, 0.4-1.6), compared with the risk for patients not undergoing a dental procedure (OR, 0.6; 95% CI, 0.4-1.1). Antibiotic prophylaxis in patients undergoing high and low risk dental procedures did not decrease the risk of prosthetic joint infections.

In 2012, the AAOS and the ADA published updated guidelines with the following summary recommendation: “The practitioner might consider discontinuing the practice of routinely prescribing prophylactic antibiotics for patients with hip and knee prosthetic joint implants undergoing dental procedures.”8 They referenced the Berbari study as the best available evidence.

Feng-Chen Kao, MD, and colleagues published a study this year with a design very similar to the Berbari study, with similar results.9 All Taiwanese residents who had received hip or knee replacements over a 12-year period were screened. Those who had received dental procedures were matched with individuals who had not had dental procedures. The dental procedure group was subdivided into a group that received antibiotics and one that didn’t.

There was no difference in infection rates between the group that had received dental procedures and the group that did not, and no difference in infection rates between those who received prophylactic antibiotics and those who didn’t.

I think this myth can be put to rest. There is no evidence to give patients with joint prostheses prophylactic antibiotics before dental procedures.


 

 

 

References

1. Steckelberg J.M., Osmon D.R. Prosthetic joint infections. In: Bisno A.L., Waldvogel F.A., eds. Infections associated with indwelling medical devices. Third ed., Washington, D.C.: American Society of Microbiology Press, 2000:173-209.

2. J Dent Res. 2004 Feb;83(2):170-4.

3. J Clin Periodontol. 2006 Feb;33(6):401-7.

4. J Am Dent Assoc. 1997 Jul;128(7):1004-8.

5. J Am Dent Assoc. 2003 Jul;134(7):895-9.

6. Spec Care Dentist. 2009 Nov-Dec;29(6):229-31.

7. Clin Infect Dis. 2010 Jan 1;50(1):8-16.

8. J Dent (Shiraz). 2013 Mar;14(1):49-52.

9. Infect Control Hosp Epidemiol. 2017 Feb;38(2):154-61.

 

Dr. Paauw is professor of medicine in the division of general internal medicine at the University of Washington, Seattle, and he serves as third-year medical student clerkship director at the University of Washington. Contact Dr. Paauw at dpaauw@uw.edu.


A 66-year-old woman 3 years status post hip replacement is seen for dental work. The dentist contacts the clinic for an antibiotic prescription. The patient has a penicillin allergy (rash). What do you recommend?

A. Clindamycin one dose before dental work.

B. Amoxicillin one dose before dental work.

C. Amoxicillin one dose before, one dose 4 hours after dental work.

D. Clindamycin one dose before dental work, one dose 4 hours after dental work.

E. No antibiotics.

Many patients with prosthetic joints will request antibiotics to take prior to dental procedures. Sometimes this request comes from the dental office.

When I ask patients why they feel they need antibiotics, they often reply that they were told by their orthopedic surgeons or their dentist that they would need to take antibiotics before dental procedures.

In an era when Clostridium difficile infection is a common and dangerous complication in the elderly, avoidance of unnecessary antibiotics is critical. In the United States, it is estimated that there are 240,000 patients infected with C. difficile annually, with 24,000 deaths at a cost of $6 billion.1

Is there compelling evidence to justify giving antibiotic prophylaxis for dental procedures to patients with prosthetic joints?

Paauw_Douglas_S_WA_web.jpg
Dr. Douglas S. Paauw
The majority of prosthetic joint infections are due to Staphylococcus aureus, whereas the majority of bacteremias from dental procedures are due to streptococcus.1,2 Bacteremias following simple everyday activities such as tooth brushing and chewing occur.3

This information has called into question the wisdom of giving antibiotic prophylaxis for dental procedures when the same patients have transient bacteremias as a regular part of day-to-day life, and mouth organisms were infrequent causes of prosthetic joint infections.

The American Academy of Orthopaedic Surgeons (AAOS) and the American Dental Association (ADA) released an advisory statement 20 years ago on antibiotic prophylaxis for patients with dental replacements, which concluded: “Antibiotic prophylaxis is not indicated for dental patients with pins, plates, and screws, nor is it routinely indicated for most dental patients with total joint replacements.”4

In 2003, the AAOS and the ADA released updated guidelines that stated: “Presently, no scientific evidence supports the position that antibiotic prophylaxis to prevent hematogenous infections is required prior to dental treatment in patients with total joint prostheses. The risk/benefit and cost/effectiveness ratios fail to justify the administration of routine antibiotics.”5

Great confusion arose in 2009 when the AAOS published a position paper on its website that reversed this position.6 Interestingly, the statement was done by the AAOS alone, and not done in conjunction with the ADA.

In this position paper, the AAOS recommended that health care providers consider antibiotic prophylaxis prior to invasive procedures on all patients who had prosthetic joints, regardless of how long those joints have been in place. This major change in recommendations was not based on any new evidence that had been reviewed since the 2003 guidelines.

There are two studies that address outcome of patients with prosthetic joints who have and have not received prophylactic antibiotics.

Elie Berbari, MD, and colleagues reported on the results of a prospective case-control study comparing patients with prosthetic joints hospitalized with hip or knee infections with patients who had prosthetic joints hospitalized at the same time who did not have hip or knee infections.7

There was no increased risk of prosthetic hip or knee infection for patients undergoing a dental procedure who were not receiving antibiotic prophylaxis (odds ratio, 0.8; 95% confidence interval, 0.4-1.6), compared with the risk for patients not undergoing a dental procedure (OR, 0.6; 95% CI, 0.4-1.1). Antibiotic prophylaxis in patients undergoing high and low risk dental procedures did not decrease the risk of prosthetic joint infections.

In 2012, the AAOS and the ADA published updated guidelines with the following summary recommendation: “The practitioner might consider discontinuing the practice of routinely prescribing prophylactic antibiotics for patients with hip and knee prosthetic joint implants undergoing dental procedures.”8 They referenced the Berbari study as the best available evidence.

Feng-Chen Kao, MD, and colleagues published a study this year with a design very similar to the Berbari study, with similar results.9 All Taiwanese residents who had received hip or knee replacements over a 12-year period were screened. Those who had received dental procedures were matched with individuals who had not had dental procedures. The dental procedure group was subdivided into a group that received antibiotics and one that didn’t.

There was no difference in infection rates between the group that had received dental procedures and the group that did not, and no difference in infection rates between those who received prophylactic antibiotics and those who didn’t.

I think this myth can be put to rest. There is no evidence to give patients with joint prostheses prophylactic antibiotics before dental procedures.


 

 

 

References

1. Steckelberg J.M., Osmon D.R. Prosthetic joint infections. In: Bisno A.L., Waldvogel F.A., eds. Infections associated with indwelling medical devices. Third ed., Washington, D.C.: American Society of Microbiology Press, 2000:173-209.

2. J Dent Res. 2004 Feb;83(2):170-4.

3. J Clin Periodontol. 2006 Feb;33(6):401-7.

4. J Am Dent Assoc. 1997 Jul;128(7):1004-8.

5. J Am Dent Assoc. 2003 Jul;134(7):895-9.

6. Spec Care Dentist. 2009 Nov-Dec;29(6):229-31.

7. Clin Infect Dis. 2010 Jan 1;50(1):8-16.

8. J Dent (Shiraz). 2013 Mar;14(1):49-52.

9. Infect Control Hosp Epidemiol. 2017 Feb;38(2):154-61.

 

Dr. Paauw is professor of medicine in the division of general internal medicine at the University of Washington, Seattle, and he serves as third-year medical student clerkship director at the University of Washington. Contact Dr. Paauw at dpaauw@uw.edu.

Publications
Publications
Topics
Article Type
Sections
Disallow All Ads
Content Gating
No Gating (article Unlocked/Free)
Alternative CME
Use ProPublica
Hide sidebar & use full width
render the right sidebar.

Myth of the Month: CT scan before lumbar puncture in suspected meningitis?

Article Type
Changed
Fri, 01/18/2019 - 16:26

 

A 28-year-old male presents to the emergency department with fever and severe headache. He has had a fever for 24 hours. Headache began that morning. On exam, he has marked nuchal rigidity, with a temperature of 103.5° F. He is fully oriented and has a nonfocal neurologic examination.

What would you do?

A) Lumbar puncture (LP).

B) CT scan, then LP.

C) Antibiotics, CT scan, then LP.

D) MRI, then LP.

E) Antibiotics, MRI, then LP.

The practice of obtaining a head CT scan (or MRI) before performing a lumbar puncture (LP) is commonplace in emergency departments. The concern is that if a lumbar puncture is done in a patient with increased intracranial pressure, then brain herniation could occur. How common is brain herniation, are there useful clinical indicators that make a CT scan not helpful, and does this concern reach myth status?

Paauw_Douglas_S_WA_web.jpg
Dr. Douglas S. Paauw
The routine use of head CT before LP in cases of suspected meningitis is common in many emergency departments.1,2 It is hard to get an absolute risk assessment on brain herniation following lumbar puncture. It does occur, but causality is hard to prove.3 Patients with rapidly developing, space-occupying lesions (hematomas, abscesses, recent large infarcts) can herniate without LP, so it is difficult to prove that herniations following LPs in case series were due to the LP.

In a retrospective study well before CT scans were available, 401 patients with brain tumors who had LP were reviewed, and 32% of the patients had papilledema.3 There was only one poor outcome because of the LP. This would be considered a high-risk group for complications, because many of these patients had clear focal neurologic signs, and one-third of them had papilledema.

There have been a number of studies looking at whether there is utility in obtaining a CT scan prior to LP in patients with suspected acute bacterial meningitis.

Dr. N.D. Baker of Brigham and Women’s Hospital, Boston, and colleagues retrospectively reviewed the records of 112 patients who had a routine CT before LP for suspected meningitis.2 Regardless of CT scan result, all patients received a lumbar puncture. Four patients had mass lesions on CT scan, though none of these patients had an increased opening pressure. No patient had an adverse outcome from LP.

Dr. Paul Greig and Dr. D. Goroszeniuk of Horton General Hospital, Banbury, England, reported on a retrospective study of all patients over a 6-month period considered for a LP.4 A total of 64 LPs were considered; 54 of these patients had a CT before LP. No patients had a bad outcome from the lumbar puncture.

The sensitivity and negative predictive value of a normal neurologic exam were very good in this study, leading to the conclusion from the authors that a normal neurologic exam and fundoscopic exam is an accurate predictor of a normal CT scan. The authors were dismayed that only 45% of the patients in the study received a fundoscopic exam.

Rodrigo Hasbun, MD, formerly of Yale University, New Haven, Conn., and his colleagues did a prospective study of 301 patients with suspected meningitis to see if clinical signs and symptoms could help guide utilization of CT scans.5 A total of 235 patients got CT scans, and 24% were abnormal.

The clinical features associated with higher likelihood of an abnormal CT were age greater than 60 years, immunosuppression, known CNS disease, and a seizure within the past week. On exam, altered mental status, aphasia, and focal neurologic findings were associated with higher likelihood of an abnormal CT scan.

Of the 96 patients who did not have any of these features, 93 had a normal CT exam, giving a negative predictive value of 97%.

No patients in the study had herniation from lumbar puncture. The only patients in the study who had brain herniations were two patients who had severe mass effect on CT, and did not receive lumbar punctures.

I think the patient in this case should have a lumbar puncture and does not need any imaging. I think that there is no reason to get neuroimaging in patients with suspected meningitis who are alert and have a nonfocal neurologic exam, and do not have papilledema.

References

1. Br J Radiol. 1999 Mar;72(855):319.

2. J Emerg Med. 1994 Sep-Oct;12(5):597-601.

3. AMA Arch Neurol Psychiatry. 1954 Nov;72(5):568-72.

4. Postgrad Med J. 2006 Mar;82(965):162-5.

5. N Engl J Med. 2001 Dec 13;345(24):1727-33.
 

Dr. Paauw is professor of medicine in the division of general internal medicine at the University of Washington, Seattle, and he serves as third-year medical student clerkship director at the University of Washington. Contact Dr. Paauw at dpaauw@uw.edu.

Publications
Topics
Sections

 

A 28-year-old male presents to the emergency department with fever and severe headache. He has had a fever for 24 hours. Headache began that morning. On exam, he has marked nuchal rigidity, with a temperature of 103.5° F. He is fully oriented and has a nonfocal neurologic examination.

What would you do?

A) Lumbar puncture (LP).

B) CT scan, then LP.

C) Antibiotics, CT scan, then LP.

D) MRI, then LP.

E) Antibiotics, MRI, then LP.

The practice of obtaining a head CT scan (or MRI) before performing a lumbar puncture (LP) is commonplace in emergency departments. The concern is that if a lumbar puncture is done in a patient with increased intracranial pressure, then brain herniation could occur. How common is brain herniation, are there useful clinical indicators that make a CT scan not helpful, and does this concern reach myth status?

Paauw_Douglas_S_WA_web.jpg
Dr. Douglas S. Paauw
The routine use of head CT before LP in cases of suspected meningitis is common in many emergency departments.1,2 It is hard to get an absolute risk assessment on brain herniation following lumbar puncture. It does occur, but causality is hard to prove.3 Patients with rapidly developing, space-occupying lesions (hematomas, abscesses, recent large infarcts) can herniate without LP, so it is difficult to prove that herniations following LPs in case series were due to the LP.

In a retrospective study well before CT scans were available, 401 patients with brain tumors who had LP were reviewed, and 32% of the patients had papilledema.3 There was only one poor outcome because of the LP. This would be considered a high-risk group for complications, because many of these patients had clear focal neurologic signs, and one-third of them had papilledema.

There have been a number of studies looking at whether there is utility in obtaining a CT scan prior to LP in patients with suspected acute bacterial meningitis.

Dr. N.D. Baker of Brigham and Women’s Hospital, Boston, and colleagues retrospectively reviewed the records of 112 patients who had a routine CT before LP for suspected meningitis.2 Regardless of CT scan result, all patients received a lumbar puncture. Four patients had mass lesions on CT scan, though none of these patients had an increased opening pressure. No patient had an adverse outcome from LP.

Dr. Paul Greig and Dr. D. Goroszeniuk of Horton General Hospital, Banbury, England, reported on a retrospective study of all patients over a 6-month period considered for a LP.4 A total of 64 LPs were considered; 54 of these patients had a CT before LP. No patients had a bad outcome from the lumbar puncture.

The sensitivity and negative predictive value of a normal neurologic exam were very good in this study, leading to the conclusion from the authors that a normal neurologic exam and fundoscopic exam is an accurate predictor of a normal CT scan. The authors were dismayed that only 45% of the patients in the study received a fundoscopic exam.

Rodrigo Hasbun, MD, formerly of Yale University, New Haven, Conn., and his colleagues did a prospective study of 301 patients with suspected meningitis to see if clinical signs and symptoms could help guide utilization of CT scans.5 A total of 235 patients got CT scans, and 24% were abnormal.

The clinical features associated with higher likelihood of an abnormal CT were age greater than 60 years, immunosuppression, known CNS disease, and a seizure within the past week. On exam, altered mental status, aphasia, and focal neurologic findings were associated with higher likelihood of an abnormal CT scan.

Of the 96 patients who did not have any of these features, 93 had a normal CT exam, giving a negative predictive value of 97%.

No patients in the study had herniation from lumbar puncture. The only patients in the study who had brain herniations were two patients who had severe mass effect on CT, and did not receive lumbar punctures.

I think the patient in this case should have a lumbar puncture and does not need any imaging. I think that there is no reason to get neuroimaging in patients with suspected meningitis who are alert and have a nonfocal neurologic exam, and do not have papilledema.

References

1. Br J Radiol. 1999 Mar;72(855):319.

2. J Emerg Med. 1994 Sep-Oct;12(5):597-601.

3. AMA Arch Neurol Psychiatry. 1954 Nov;72(5):568-72.

4. Postgrad Med J. 2006 Mar;82(965):162-5.

5. N Engl J Med. 2001 Dec 13;345(24):1727-33.
 

Dr. Paauw is professor of medicine in the division of general internal medicine at the University of Washington, Seattle, and he serves as third-year medical student clerkship director at the University of Washington. Contact Dr. Paauw at dpaauw@uw.edu.

 

A 28-year-old male presents to the emergency department with fever and severe headache. He has had a fever for 24 hours. Headache began that morning. On exam, he has marked nuchal rigidity, with a temperature of 103.5° F. He is fully oriented and has a nonfocal neurologic examination.

What would you do?

A) Lumbar puncture (LP).

B) CT scan, then LP.

C) Antibiotics, CT scan, then LP.

D) MRI, then LP.

E) Antibiotics, MRI, then LP.

The practice of obtaining a head CT scan (or MRI) before performing a lumbar puncture (LP) is commonplace in emergency departments. The concern is that if a lumbar puncture is done in a patient with increased intracranial pressure, then brain herniation could occur. How common is brain herniation, are there useful clinical indicators that make a CT scan not helpful, and does this concern reach myth status?

Paauw_Douglas_S_WA_web.jpg
Dr. Douglas S. Paauw
The routine use of head CT before LP in cases of suspected meningitis is common in many emergency departments.1,2 It is hard to get an absolute risk assessment on brain herniation following lumbar puncture. It does occur, but causality is hard to prove.3 Patients with rapidly developing, space-occupying lesions (hematomas, abscesses, recent large infarcts) can herniate without LP, so it is difficult to prove that herniations following LPs in case series were due to the LP.

In a retrospective study well before CT scans were available, 401 patients with brain tumors who had LP were reviewed, and 32% of the patients had papilledema.3 There was only one poor outcome because of the LP. This would be considered a high-risk group for complications, because many of these patients had clear focal neurologic signs, and one-third of them had papilledema.

There have been a number of studies looking at whether there is utility in obtaining a CT scan prior to LP in patients with suspected acute bacterial meningitis.

Dr. N.D. Baker of Brigham and Women’s Hospital, Boston, and colleagues retrospectively reviewed the records of 112 patients who had a routine CT before LP for suspected meningitis.2 Regardless of CT scan result, all patients received a lumbar puncture. Four patients had mass lesions on CT scan, though none of these patients had an increased opening pressure. No patient had an adverse outcome from LP.

Dr. Paul Greig and Dr. D. Goroszeniuk of Horton General Hospital, Banbury, England, reported on a retrospective study of all patients over a 6-month period considered for a LP.4 A total of 64 LPs were considered; 54 of these patients had a CT before LP. No patients had a bad outcome from the lumbar puncture.

The sensitivity and negative predictive value of a normal neurologic exam were very good in this study, leading to the conclusion from the authors that a normal neurologic exam and fundoscopic exam is an accurate predictor of a normal CT scan. The authors were dismayed that only 45% of the patients in the study received a fundoscopic exam.

Rodrigo Hasbun, MD, formerly of Yale University, New Haven, Conn., and his colleagues did a prospective study of 301 patients with suspected meningitis to see if clinical signs and symptoms could help guide utilization of CT scans.5 A total of 235 patients got CT scans, and 24% were abnormal.

The clinical features associated with higher likelihood of an abnormal CT were age greater than 60 years, immunosuppression, known CNS disease, and a seizure within the past week. On exam, altered mental status, aphasia, and focal neurologic findings were associated with higher likelihood of an abnormal CT scan.

Of the 96 patients who did not have any of these features, 93 had a normal CT exam, giving a negative predictive value of 97%.

No patients in the study had herniation from lumbar puncture. The only patients in the study who had brain herniations were two patients who had severe mass effect on CT, and did not receive lumbar punctures.

I think the patient in this case should have a lumbar puncture and does not need any imaging. I think that there is no reason to get neuroimaging in patients with suspected meningitis who are alert and have a nonfocal neurologic exam, and do not have papilledema.

References

1. Br J Radiol. 1999 Mar;72(855):319.

2. J Emerg Med. 1994 Sep-Oct;12(5):597-601.

3. AMA Arch Neurol Psychiatry. 1954 Nov;72(5):568-72.

4. Postgrad Med J. 2006 Mar;82(965):162-5.

5. N Engl J Med. 2001 Dec 13;345(24):1727-33.
 

Dr. Paauw is professor of medicine in the division of general internal medicine at the University of Washington, Seattle, and he serves as third-year medical student clerkship director at the University of Washington. Contact Dr. Paauw at dpaauw@uw.edu.

Publications
Publications
Topics
Article Type
Sections
Disallow All Ads
Content Gating
No Gating (article Unlocked/Free)

Myth of the Month: Does nitroglycerin response predict coronary artery disease?

Article Type
Changed
Fri, 01/18/2019 - 16:17

 

A 55-year-old man presents to the emergency department with substernal chest pain. The pain has occurred off and on over the past 2 hours. He has no family history of coronary artery disease. He has no history of diabetes, hypertension, or cigarette smoking. His most recent total cholesterol was 220 mg/dL (HDL, 40; LDL, 155). Blood pressure is 130/70. An ECG obtained on arrival is unremarkable. When he reached the ED, he received a nitroglycerin tablet with resolution of his pain within 4 minutes.

What is the most accurate statement?

A. The chance of CAD in this man over the next 10 years was 8% before his symptoms and is now greater than 20%.

B. The chance of CAD in this man over the next 10 years was 8% and is still 8%.

C. The chance of CAD in this man over the next 10 years was 15% before his symptoms and is now close to 100%.

D. The chance of CAD in this man over the next 10 years was 15% before his symptoms and is now close to 50%.

For years, giving nitroglycerin to patients who present with chest pain has been considered a good therapy, and the response to the medication has been considered a sign that the pain was likely due to cardiac ischemia. Is there evidence that this is true?

Paauw_Douglas_S_WA_web.jpg
Dr. Douglas S. Paauw
Eric A. Shry, MD, and his colleagues looked at the usefulness of nitroglycerin response in the treatment of chest pain as a predictor of ischemic chest pain in an emergency department setting.1

The study was a retrospective review of 223 patients who presented to the ED over a 5-month period with ongoing chest pain. They looked at patients who had ongoing chest pain in the ED, received nitroglycerin, and did not receive any therapy other than aspirin within 10 minutes of receiving nitroglycerin. Nitroglycerin response was compared with the final diagnosis of cardiac versus noncardiac chest pain.

Of the patients with a final determination of cardiac chest pain, 88% had a nitroglycerin response, whereas 92% of the patients with noncardiac chest pain had a nitroglycerin response (P = .50).

Deborah B. Diercks, MD, and her colleagues looked at improvement in chest pain scores in the ED in patients treated with nitroglycerin and whether it correlated with a cardiac etiology of chest pain.2 The study was a prospective, observational study of 664 patients in an urban tertiary care ED over a 16-month period. An 11-point numeric chest pain scale was assessed and recorded by research assistants before and 5 minutes after receiving nitroglycerin. The scale ranged from 0 (no pain) to 10 (worst pain imaginable).

A final diagnosis of a cardiac etiology for chest pain was found in 18% of the patients in the study. Of the patients who had cardiac-related chest pain, 20% had no reduction in pain with nitroglycerin, compared with 19% of the patients without cardiac-related chest pain. Complete or significant reduction in chest pain occurred with nitroglycerin in 31% of patients with cardiac chest pain and 27% of the patients without cardiac chest pain (P = .76).

Two other studies with similar designs showed similar results. Robert Steele, MD, and his colleagues studied 270 patients in a prospective observational cohort study of patients with chest pain presenting to an urban ED.3 Patients presenting to the ED with active chest pain who received nitroglycerin were enrolled.

The sensitivity in this study for nitroglycerin relief determining cardiac chest pain was 72%, and the specificity was 37%, with a positive likelihood ratio for coronary artery disease if nitroglycerin response of 1.1 (0.96-1.34).

In another prospective, observational cohort study, 459 patients who presented to an ED with chest pain were evaluated for response to nitroglycerin as a marker for ischemic cardiac disease.4 In this study, presence of ischemic cardiac disease was defined as diagnosis in the ED or during a 4-month follow-up period. Nitroglycerin relieved chest pain in 35% of patients who had coronary disease, whereas 41% of patients without coronary disease had a nitroglycerin response. This study had a much lower overall nitroglycerin response rate than any of the other studies.

Katherine Grailey, MD, and Paul Glasziou, MD, PhD, published a meta-analysis of nitroglycerin use for the diagnosis of chest pain, using the above referenced studies. They concluded that in the acute setting, nitroglycerin is not a reliable test of treatment for use in diagnosis of coronary artery disease.5

High response rate for nitroglycerin in the noncoronary artery groups in the studies may be due to a strong placebo effect and/or that nitroglycerin may help with pain caused by esophageal spasm. The lack of specificity in the pain relief response for nitroglycerin makes it not a helpful test. Note that all the studies have been in the acute, ED setting for chest pain. In the case presented at the beginning of the article, the response the patient had to nitroglycerin would not change the probability that he has coronary artery disease.
 

 

 

References

1. Am J Cardiol. 2002 Dec 1;90(11):1264-6.

2. Ann Emerg Med. 2005 Jun;45(6):581-5.

3. CJEM. 2006 May;8(3):164-9.

4. Ann Intern Med. 2003 Dec 16;139(12):979-86.

5. Emerg Med J. 2012 Mar;29(3):173-6.

Dr. Paauw is professor of medicine in the division of general internal medicine at the University of Washington, Seattle, and he serves as third-year medical student clerkship director at the University of Washington. Contact Dr. Paauw at dpaauw@uw.edu .

Issue
Emergency Medicine - 12(6)
Publications
Topics
Sections

 

A 55-year-old man presents to the emergency department with substernal chest pain. The pain has occurred off and on over the past 2 hours. He has no family history of coronary artery disease. He has no history of diabetes, hypertension, or cigarette smoking. His most recent total cholesterol was 220 mg/dL (HDL, 40; LDL, 155). Blood pressure is 130/70. An ECG obtained on arrival is unremarkable. When he reached the ED, he received a nitroglycerin tablet with resolution of his pain within 4 minutes.

What is the most accurate statement?

A. The chance of CAD in this man over the next 10 years was 8% before his symptoms and is now greater than 20%.

B. The chance of CAD in this man over the next 10 years was 8% and is still 8%.

C. The chance of CAD in this man over the next 10 years was 15% before his symptoms and is now close to 100%.

D. The chance of CAD in this man over the next 10 years was 15% before his symptoms and is now close to 50%.

For years, giving nitroglycerin to patients who present with chest pain has been considered a good therapy, and the response to the medication has been considered a sign that the pain was likely due to cardiac ischemia. Is there evidence that this is true?

Paauw_Douglas_S_WA_web.jpg
Dr. Douglas S. Paauw
Eric A. Shry, MD, and his colleagues looked at the usefulness of nitroglycerin response in the treatment of chest pain as a predictor of ischemic chest pain in an emergency department setting.1

The study was a retrospective review of 223 patients who presented to the ED over a 5-month period with ongoing chest pain. They looked at patients who had ongoing chest pain in the ED, received nitroglycerin, and did not receive any therapy other than aspirin within 10 minutes of receiving nitroglycerin. Nitroglycerin response was compared with the final diagnosis of cardiac versus noncardiac chest pain.

Of the patients with a final determination of cardiac chest pain, 88% had a nitroglycerin response, whereas 92% of the patients with noncardiac chest pain had a nitroglycerin response (P = .50).

Deborah B. Diercks, MD, and her colleagues looked at improvement in chest pain scores in the ED in patients treated with nitroglycerin and whether it correlated with a cardiac etiology of chest pain.2 The study was a prospective, observational study of 664 patients in an urban tertiary care ED over a 16-month period. An 11-point numeric chest pain scale was assessed and recorded by research assistants before and 5 minutes after receiving nitroglycerin. The scale ranged from 0 (no pain) to 10 (worst pain imaginable).

A final diagnosis of a cardiac etiology for chest pain was found in 18% of the patients in the study. Of the patients who had cardiac-related chest pain, 20% had no reduction in pain with nitroglycerin, compared with 19% of the patients without cardiac-related chest pain. Complete or significant reduction in chest pain occurred with nitroglycerin in 31% of patients with cardiac chest pain and 27% of the patients without cardiac chest pain (P = .76).

Two other studies with similar designs showed similar results. Robert Steele, MD, and his colleagues studied 270 patients in a prospective observational cohort study of patients with chest pain presenting to an urban ED.3 Patients presenting to the ED with active chest pain who received nitroglycerin were enrolled.

The sensitivity in this study for nitroglycerin relief determining cardiac chest pain was 72%, and the specificity was 37%, with a positive likelihood ratio for coronary artery disease if nitroglycerin response of 1.1 (0.96-1.34).

In another prospective, observational cohort study, 459 patients who presented to an ED with chest pain were evaluated for response to nitroglycerin as a marker for ischemic cardiac disease.4 In this study, presence of ischemic cardiac disease was defined as diagnosis in the ED or during a 4-month follow-up period. Nitroglycerin relieved chest pain in 35% of patients who had coronary disease, whereas 41% of patients without coronary disease had a nitroglycerin response. This study had a much lower overall nitroglycerin response rate than any of the other studies.

Katherine Grailey, MD, and Paul Glasziou, MD, PhD, published a meta-analysis of nitroglycerin use for the diagnosis of chest pain, using the above referenced studies. They concluded that in the acute setting, nitroglycerin is not a reliable test of treatment for use in diagnosis of coronary artery disease.5

High response rate for nitroglycerin in the noncoronary artery groups in the studies may be due to a strong placebo effect and/or that nitroglycerin may help with pain caused by esophageal spasm. The lack of specificity in the pain relief response for nitroglycerin makes it not a helpful test. Note that all the studies have been in the acute, ED setting for chest pain. In the case presented at the beginning of the article, the response the patient had to nitroglycerin would not change the probability that he has coronary artery disease.
 

 

 

References

1. Am J Cardiol. 2002 Dec 1;90(11):1264-6.

2. Ann Emerg Med. 2005 Jun;45(6):581-5.

3. CJEM. 2006 May;8(3):164-9.

4. Ann Intern Med. 2003 Dec 16;139(12):979-86.

5. Emerg Med J. 2012 Mar;29(3):173-6.

Dr. Paauw is professor of medicine in the division of general internal medicine at the University of Washington, Seattle, and he serves as third-year medical student clerkship director at the University of Washington. Contact Dr. Paauw at dpaauw@uw.edu .

 

A 55-year-old man presents to the emergency department with substernal chest pain. The pain has occurred off and on over the past 2 hours. He has no family history of coronary artery disease. He has no history of diabetes, hypertension, or cigarette smoking. His most recent total cholesterol was 220 mg/dL (HDL, 40; LDL, 155). Blood pressure is 130/70. An ECG obtained on arrival is unremarkable. When he reached the ED, he received a nitroglycerin tablet with resolution of his pain within 4 minutes.

What is the most accurate statement?

A. The chance of CAD in this man over the next 10 years was 8% before his symptoms and is now greater than 20%.

B. The chance of CAD in this man over the next 10 years was 8% and is still 8%.

C. The chance of CAD in this man over the next 10 years was 15% before his symptoms and is now close to 100%.

D. The chance of CAD in this man over the next 10 years was 15% before his symptoms and is now close to 50%.

For years, giving nitroglycerin to patients who present with chest pain has been considered a good therapy, and the response to the medication has been considered a sign that the pain was likely due to cardiac ischemia. Is there evidence that this is true?

Paauw_Douglas_S_WA_web.jpg
Dr. Douglas S. Paauw
Eric A. Shry, MD, and his colleagues looked at the usefulness of nitroglycerin response in the treatment of chest pain as a predictor of ischemic chest pain in an emergency department setting.1

The study was a retrospective review of 223 patients who presented to the ED over a 5-month period with ongoing chest pain. They looked at patients who had ongoing chest pain in the ED, received nitroglycerin, and did not receive any therapy other than aspirin within 10 minutes of receiving nitroglycerin. Nitroglycerin response was compared with the final diagnosis of cardiac versus noncardiac chest pain.

Of the patients with a final determination of cardiac chest pain, 88% had a nitroglycerin response, whereas 92% of the patients with noncardiac chest pain had a nitroglycerin response (P = .50).

Deborah B. Diercks, MD, and her colleagues looked at improvement in chest pain scores in the ED in patients treated with nitroglycerin and whether it correlated with a cardiac etiology of chest pain.2 The study was a prospective, observational study of 664 patients in an urban tertiary care ED over a 16-month period. An 11-point numeric chest pain scale was assessed and recorded by research assistants before and 5 minutes after receiving nitroglycerin. The scale ranged from 0 (no pain) to 10 (worst pain imaginable).

A final diagnosis of a cardiac etiology for chest pain was found in 18% of the patients in the study. Of the patients who had cardiac-related chest pain, 20% had no reduction in pain with nitroglycerin, compared with 19% of the patients without cardiac-related chest pain. Complete or significant reduction in chest pain occurred with nitroglycerin in 31% of patients with cardiac chest pain and 27% of the patients without cardiac chest pain (P = .76).

Two other studies with similar designs showed similar results. Robert Steele, MD, and his colleagues studied 270 patients in a prospective observational cohort study of patients with chest pain presenting to an urban ED.3 Patients presenting to the ED with active chest pain who received nitroglycerin were enrolled.

The sensitivity in this study for nitroglycerin relief determining cardiac chest pain was 72%, and the specificity was 37%, with a positive likelihood ratio for coronary artery disease if nitroglycerin response of 1.1 (0.96-1.34).

In another prospective, observational cohort study, 459 patients who presented to an ED with chest pain were evaluated for response to nitroglycerin as a marker for ischemic cardiac disease.4 In this study, presence of ischemic cardiac disease was defined as diagnosis in the ED or during a 4-month follow-up period. Nitroglycerin relieved chest pain in 35% of patients who had coronary disease, whereas 41% of patients without coronary disease had a nitroglycerin response. This study had a much lower overall nitroglycerin response rate than any of the other studies.

Katherine Grailey, MD, and Paul Glasziou, MD, PhD, published a meta-analysis of nitroglycerin use for the diagnosis of chest pain, using the above referenced studies. They concluded that in the acute setting, nitroglycerin is not a reliable test of treatment for use in diagnosis of coronary artery disease.5

High response rate for nitroglycerin in the noncoronary artery groups in the studies may be due to a strong placebo effect and/or that nitroglycerin may help with pain caused by esophageal spasm. The lack of specificity in the pain relief response for nitroglycerin makes it not a helpful test. Note that all the studies have been in the acute, ED setting for chest pain. In the case presented at the beginning of the article, the response the patient had to nitroglycerin would not change the probability that he has coronary artery disease.
 

 

 

References

1. Am J Cardiol. 2002 Dec 1;90(11):1264-6.

2. Ann Emerg Med. 2005 Jun;45(6):581-5.

3. CJEM. 2006 May;8(3):164-9.

4. Ann Intern Med. 2003 Dec 16;139(12):979-86.

5. Emerg Med J. 2012 Mar;29(3):173-6.

Dr. Paauw is professor of medicine in the division of general internal medicine at the University of Washington, Seattle, and he serves as third-year medical student clerkship director at the University of Washington. Contact Dr. Paauw at dpaauw@uw.edu .

Issue
Emergency Medicine - 12(6)
Issue
Emergency Medicine - 12(6)
Publications
Publications
Topics
Article Type
Sections
Disallow All Ads

Myth of the Month: Vaccinations in patients with Guillain-Barré syndrome

Article Type
Changed
Fri, 01/18/2019 - 16:08
Display Headline
Myth of the Month: Vaccinations in patients with Guillain-Barré syndrome

A 66-year-old woman presents as a new patient for a clinic visit. She has a history of Guillain-Barré syndrome 10 years ago. The last immunization she received was a tetanus-diphtheria 12 years ago.

What do you recommend for her to receive over the next year?

A. Pneumococcal 13/Pneumococcal 23/Tdap/influenza vaccines.

B. Pneumococcal 13/Pneumococcal 23/Tdap vaccines.

C. Influenza vaccine.

D. No vaccines.

 

RTEmagicC_673cdb1e2806674d2c_Paauw_Douglas_S_WA.jpg.jpg
Dr. Douglas S. Paauw

Guillain-Barré syndrome (GBS) is a rare, acute, immune-mediated polyneuropathy that has an incidence of about 2 cases per 100,000 people each year.1 Most cases of GBS follow an infectious event (usually an upper respiratory infection or gastrointestinal infection). In 1976, administration of the swine flu vaccine was associated with an up to eightfold increased risk of GBS.2,3 Many patients who have had GBS have been advised not to – or are fearful to – receive influenza vaccine or any vaccine.

Is there good evidence for patients with a history of GBS to avoid influenza vaccines or vaccinations in general?

The initial concern over the increased risk of GBS following the large-scale influenza vaccination in 1976 has not been realized with subsequent influenza vaccines. In a study by Baxter and colleagues, GBS cases from Kaiser Permanente Northern California from 1995 to 2006 were reviewed.4 They looked at whether patients had received influenza vaccine in the 6 weeks prior to GBS, compared with vaccination within the prior 9 months.

The odds ratio for influenza vaccination in the 6 weeks prior to GBS was 1.1 (95% confidence interval, 0.4-3.1). The odds ratio for receiving tetanus diphtheria vaccine in the 6 weeks prior to GBS was 1.4 (95% CI, 0.3-4.5); pneumococcal 23 vaccine, 0.7 (95% CI, 0.1-2.9); and all vaccines combined, 1.3 (95% CI, 0.8-2.3).

Shahed Iqbal, MBBS, et al. looked at the relationship between influenza illness, pneumonia, influenza vaccination, and GBS.5 They found that although influenza vaccine coverage increased from 20% to 36% over the study period, there was not an increase in GBS hospitalizations over the same period. There was a significant correlation between hospitalizations for pneumonia and influenza and GBS hospitalizations in the same month.

In a simulation study, Steven Hawken, PhD, and his colleagues concluded that under typical conditions (influenza incidence greater than 5% and vaccine effectiveness greater than 60%), influenza vaccination reduced GBS risk.6

There are fewer data on vaccination in patients who have previously had GBS, but there is enough evidence to help guide us.

Roger Baxter, MD, and colleagues, using the database in reference 4, looked at outcome of patients with GBS who received vaccinations subsequent to recovery from GBS.7 A total of 279 patient with previous GBS received a total of 989 vaccinations, including 405 trivalent influenza vaccinations. None of the patients with GBS who received vaccinations had a recurrence of GBS.

Krista Kuitwaard, MD, et al. reported identical findings in a survey of patients with a history of GBS or chronic inflammatory demyelinating polyradiculoneuropathy (CIDP).8 A total of 245 patients with GBS responded to the survey. A total of 106 GBS patients had received influenza vaccine following their GBS diagnosis (a total of 775 vaccinations in those patients). None of the patients with a history of GBS who received influenza vaccination had a recurrence of their GBS.

The current position of the GBS/CIDP Foundation on vaccination for patients with GBS is as follows: The GBS/CIDP Foundation recommends avoiding immunizations that a GBS patient had received within 6 weeks of developing their initial symptoms.9

I think the current evidence is enough to guide us in this issue. Vaccinations, including influenza vaccine, are likely safe for patients with a history of GBS. The recommendation of the GBS/CIDP foundation is reasonable – to avoid immunizations that appeared to have potentially triggered the initial GBS (ones that had been received within 6 weeks of onset of symptoms).

In the case presented above, I think that choice A – receiving all the recommended immunizations – would be appropriate.

References

1. Neuroepidemiology 2011; 36(2):123-33.

2. Am J Epidemiol. 1979 Aug;110(2):105-23.

3. Clin Infect Dis. 2014 Apr;58(8):1149-55.

4. Clin Infect Dis. 2013 Jul;57(2):197-204.

5. Vaccine. 2015 Apr 21;33(17):2045-9.

6. Emerg Infect Dis. 2015 Feb;21(2):224-31.

7. Clin Infect Dis. 2012 Mar;54(6):800-4.

8. J Peripher Nerv Syst. 2009 Dec;14(4):310-5.

9. GBS/CIDP Foundation International, Position on Flu Shots and Vaccinations.

Dr. Paauw is professor of medicine in the division of general internal medicine at the University of Washington, Seattle, and he serves as third-year medical student clerkship director at the University of Washington. Contact Dr. Paauw at dpaauw@uw.edu.

Publications
Topics
Legacy Keywords
Guillain-Barré syndrome, GBS, vaccine, vaccination, immunization, flu shot, influenza, TDAP
Sections

A 66-year-old woman presents as a new patient for a clinic visit. She has a history of Guillain-Barré syndrome 10 years ago. The last immunization she received was a tetanus-diphtheria 12 years ago.

What do you recommend for her to receive over the next year?

A. Pneumococcal 13/Pneumococcal 23/Tdap/influenza vaccines.

B. Pneumococcal 13/Pneumococcal 23/Tdap vaccines.

C. Influenza vaccine.

D. No vaccines.

 

RTEmagicC_673cdb1e2806674d2c_Paauw_Douglas_S_WA.jpg.jpg
Dr. Douglas S. Paauw

Guillain-Barré syndrome (GBS) is a rare, acute, immune-mediated polyneuropathy that has an incidence of about 2 cases per 100,000 people each year.1 Most cases of GBS follow an infectious event (usually an upper respiratory infection or gastrointestinal infection). In 1976, administration of the swine flu vaccine was associated with an up to eightfold increased risk of GBS.2,3 Many patients who have had GBS have been advised not to – or are fearful to – receive influenza vaccine or any vaccine.

Is there good evidence for patients with a history of GBS to avoid influenza vaccines or vaccinations in general?

The initial concern over the increased risk of GBS following the large-scale influenza vaccination in 1976 has not been realized with subsequent influenza vaccines. In a study by Baxter and colleagues, GBS cases from Kaiser Permanente Northern California from 1995 to 2006 were reviewed.4 They looked at whether patients had received influenza vaccine in the 6 weeks prior to GBS, compared with vaccination within the prior 9 months.

The odds ratio for influenza vaccination in the 6 weeks prior to GBS was 1.1 (95% confidence interval, 0.4-3.1). The odds ratio for receiving tetanus diphtheria vaccine in the 6 weeks prior to GBS was 1.4 (95% CI, 0.3-4.5); pneumococcal 23 vaccine, 0.7 (95% CI, 0.1-2.9); and all vaccines combined, 1.3 (95% CI, 0.8-2.3).

Shahed Iqbal, MBBS, et al. looked at the relationship between influenza illness, pneumonia, influenza vaccination, and GBS.5 They found that although influenza vaccine coverage increased from 20% to 36% over the study period, there was not an increase in GBS hospitalizations over the same period. There was a significant correlation between hospitalizations for pneumonia and influenza and GBS hospitalizations in the same month.

In a simulation study, Steven Hawken, PhD, and his colleagues concluded that under typical conditions (influenza incidence greater than 5% and vaccine effectiveness greater than 60%), influenza vaccination reduced GBS risk.6

There are fewer data on vaccination in patients who have previously had GBS, but there is enough evidence to help guide us.

Roger Baxter, MD, and colleagues, using the database in reference 4, looked at outcome of patients with GBS who received vaccinations subsequent to recovery from GBS.7 A total of 279 patient with previous GBS received a total of 989 vaccinations, including 405 trivalent influenza vaccinations. None of the patients with GBS who received vaccinations had a recurrence of GBS.

Krista Kuitwaard, MD, et al. reported identical findings in a survey of patients with a history of GBS or chronic inflammatory demyelinating polyradiculoneuropathy (CIDP).8 A total of 245 patients with GBS responded to the survey. A total of 106 GBS patients had received influenza vaccine following their GBS diagnosis (a total of 775 vaccinations in those patients). None of the patients with a history of GBS who received influenza vaccination had a recurrence of their GBS.

The current position of the GBS/CIDP Foundation on vaccination for patients with GBS is as follows: The GBS/CIDP Foundation recommends avoiding immunizations that a GBS patient had received within 6 weeks of developing their initial symptoms.9

I think the current evidence is enough to guide us in this issue. Vaccinations, including influenza vaccine, are likely safe for patients with a history of GBS. The recommendation of the GBS/CIDP foundation is reasonable – to avoid immunizations that appeared to have potentially triggered the initial GBS (ones that had been received within 6 weeks of onset of symptoms).

In the case presented above, I think that choice A – receiving all the recommended immunizations – would be appropriate.

References

1. Neuroepidemiology 2011; 36(2):123-33.

2. Am J Epidemiol. 1979 Aug;110(2):105-23.

3. Clin Infect Dis. 2014 Apr;58(8):1149-55.

4. Clin Infect Dis. 2013 Jul;57(2):197-204.

5. Vaccine. 2015 Apr 21;33(17):2045-9.

6. Emerg Infect Dis. 2015 Feb;21(2):224-31.

7. Clin Infect Dis. 2012 Mar;54(6):800-4.

8. J Peripher Nerv Syst. 2009 Dec;14(4):310-5.

9. GBS/CIDP Foundation International, Position on Flu Shots and Vaccinations.

Dr. Paauw is professor of medicine in the division of general internal medicine at the University of Washington, Seattle, and he serves as third-year medical student clerkship director at the University of Washington. Contact Dr. Paauw at dpaauw@uw.edu.

A 66-year-old woman presents as a new patient for a clinic visit. She has a history of Guillain-Barré syndrome 10 years ago. The last immunization she received was a tetanus-diphtheria 12 years ago.

What do you recommend for her to receive over the next year?

A. Pneumococcal 13/Pneumococcal 23/Tdap/influenza vaccines.

B. Pneumococcal 13/Pneumococcal 23/Tdap vaccines.

C. Influenza vaccine.

D. No vaccines.

 

RTEmagicC_673cdb1e2806674d2c_Paauw_Douglas_S_WA.jpg.jpg
Dr. Douglas S. Paauw

Guillain-Barré syndrome (GBS) is a rare, acute, immune-mediated polyneuropathy that has an incidence of about 2 cases per 100,000 people each year.1 Most cases of GBS follow an infectious event (usually an upper respiratory infection or gastrointestinal infection). In 1976, administration of the swine flu vaccine was associated with an up to eightfold increased risk of GBS.2,3 Many patients who have had GBS have been advised not to – or are fearful to – receive influenza vaccine or any vaccine.

Is there good evidence for patients with a history of GBS to avoid influenza vaccines or vaccinations in general?

The initial concern over the increased risk of GBS following the large-scale influenza vaccination in 1976 has not been realized with subsequent influenza vaccines. In a study by Baxter and colleagues, GBS cases from Kaiser Permanente Northern California from 1995 to 2006 were reviewed.4 They looked at whether patients had received influenza vaccine in the 6 weeks prior to GBS, compared with vaccination within the prior 9 months.

The odds ratio for influenza vaccination in the 6 weeks prior to GBS was 1.1 (95% confidence interval, 0.4-3.1). The odds ratio for receiving tetanus diphtheria vaccine in the 6 weeks prior to GBS was 1.4 (95% CI, 0.3-4.5); pneumococcal 23 vaccine, 0.7 (95% CI, 0.1-2.9); and all vaccines combined, 1.3 (95% CI, 0.8-2.3).

Shahed Iqbal, MBBS, et al. looked at the relationship between influenza illness, pneumonia, influenza vaccination, and GBS.5 They found that although influenza vaccine coverage increased from 20% to 36% over the study period, there was not an increase in GBS hospitalizations over the same period. There was a significant correlation between hospitalizations for pneumonia and influenza and GBS hospitalizations in the same month.

In a simulation study, Steven Hawken, PhD, and his colleagues concluded that under typical conditions (influenza incidence greater than 5% and vaccine effectiveness greater than 60%), influenza vaccination reduced GBS risk.6

There are fewer data on vaccination in patients who have previously had GBS, but there is enough evidence to help guide us.

Roger Baxter, MD, and colleagues, using the database in reference 4, looked at outcome of patients with GBS who received vaccinations subsequent to recovery from GBS.7 A total of 279 patient with previous GBS received a total of 989 vaccinations, including 405 trivalent influenza vaccinations. None of the patients with GBS who received vaccinations had a recurrence of GBS.

Krista Kuitwaard, MD, et al. reported identical findings in a survey of patients with a history of GBS or chronic inflammatory demyelinating polyradiculoneuropathy (CIDP).8 A total of 245 patients with GBS responded to the survey. A total of 106 GBS patients had received influenza vaccine following their GBS diagnosis (a total of 775 vaccinations in those patients). None of the patients with a history of GBS who received influenza vaccination had a recurrence of their GBS.

The current position of the GBS/CIDP Foundation on vaccination for patients with GBS is as follows: The GBS/CIDP Foundation recommends avoiding immunizations that a GBS patient had received within 6 weeks of developing their initial symptoms.9

I think the current evidence is enough to guide us in this issue. Vaccinations, including influenza vaccine, are likely safe for patients with a history of GBS. The recommendation of the GBS/CIDP foundation is reasonable – to avoid immunizations that appeared to have potentially triggered the initial GBS (ones that had been received within 6 weeks of onset of symptoms).

In the case presented above, I think that choice A – receiving all the recommended immunizations – would be appropriate.

References

1. Neuroepidemiology 2011; 36(2):123-33.

2. Am J Epidemiol. 1979 Aug;110(2):105-23.

3. Clin Infect Dis. 2014 Apr;58(8):1149-55.

4. Clin Infect Dis. 2013 Jul;57(2):197-204.

5. Vaccine. 2015 Apr 21;33(17):2045-9.

6. Emerg Infect Dis. 2015 Feb;21(2):224-31.

7. Clin Infect Dis. 2012 Mar;54(6):800-4.

8. J Peripher Nerv Syst. 2009 Dec;14(4):310-5.

9. GBS/CIDP Foundation International, Position on Flu Shots and Vaccinations.

Dr. Paauw is professor of medicine in the division of general internal medicine at the University of Washington, Seattle, and he serves as third-year medical student clerkship director at the University of Washington. Contact Dr. Paauw at dpaauw@uw.edu.

Publications
Publications
Topics
Article Type
Display Headline
Myth of the Month: Vaccinations in patients with Guillain-Barré syndrome
Display Headline
Myth of the Month: Vaccinations in patients with Guillain-Barré syndrome
Legacy Keywords
Guillain-Barré syndrome, GBS, vaccine, vaccination, immunization, flu shot, influenza, TDAP
Legacy Keywords
Guillain-Barré syndrome, GBS, vaccine, vaccination, immunization, flu shot, influenza, TDAP
Sections
Disallow All Ads

Myth of the Month: NPO good for people with pancreatitis?

Article Type
Changed
Fri, 01/18/2019 - 15:55
Display Headline
Myth of the Month: NPO good for people with pancreatitis?

A 60-year-old man presents to the emergency department with nausea and abdominal pain, and is admitted with pancreatitis due to alcohol. In the evening after receiving pain medication, his abdominal pain is diminished but still present. He has an appetite and asks for food.

What do you recommend?

A. Nil per os (NPO) until pain is resolved.

B. NPO until amylase/lipase have normalized.

C. Nasogastric tube placement.

D. Okay to start feeding.

Myth: Treatment of pancreatitis includes early avoidance of food.

 

RTEmagicC_2d37f4d571531c22fc_Paauw_Douglas_S_WA.jpg.jpg
Dr. Douglas S. Paauw

The conventional management of acute pancreatitis involves an NPO regimen until the pain and nausea have resolved.1 This dogma is offered because of the concern that food intake will stimulate pancreatic enzyme release in an already inflamed/injured pancreas.

The approach of NPO and slowly reintroducing feeding after prolonged periods of being without food is associated with pain relapses and increased length of hospitalizations.2 Nasojejunal feedings have become well accepted in patients with severe pancreatitis requiring ICU care.3

Are there data to show that oral feeding of patients with mild pancreatitis causes worse outcomes?

Dr. Niels Teich and colleagues randomized 143 hospitalized patients with mild pancreatitis to eating when they felt ready to (69 patients) vs. a group that were kept NPO until lipase levels returned to normal.4 The patients who started eating when they were ready left the hospital a day earlier than the patients who were fed only when lipase levels normalized (7 days vs. 8 days). There was no difference in abdominal pain between the two groups.

Dr. Maxim Petrov and colleagues looked at whether nasogastric tube feeding was preferable to NPO in patients with mild to moderate pancreatitis.5 In a randomized trial of 35 patients with pancreatitis, 17 received nasogastric feedings within 24 hours of admission, and 18 were NPO. The patients who received early nasogastric feedings had lower pain scores at 72 hours, compared with the NPO group (1 vs. 3 on a visual analog 10-point scale, P = .036). The number of patients who did not require opiates at 48 hours was also significantly less in the nasogastric feeding group (9 vs. 3, P = .024).

I think the most striking difference was in patients’ ability to tolerate oral feeding. Patients in both groups received oral food at an average of 4 days; only 1 of 17 patients in the nasogastric feeding group could not tolerate feeding, compared with 9 of 18 patients in the NPO group.

Dr. Gunilla Eckerwall and colleagues studied the outcome of immediate oral feeding in patients with mild pancreatitis.6 Sixty patients with mild acute pancreatitis, defined by amylase greater than 3 times normal and APACHE scores less than 8, were randomized to either immediate oral feeding (30 patients) or fasting (30 patients). Key outcome measures in the study were amylase, systemic inflammatory response, and length of hospital stay.

There were no differences in amylase levels, labs measuring systemic inflammatory response, or gastrointestinal symptoms between the two groups. The immediate oral feeding group had a significantly shorter length of hospital stay than the fasting group (4 days vs. 6 days, P less than .05).

So, what does all this tell us about feeding patients with acute pancreatitis? For mild to moderate acute pancreatitis, the outcomes appear to be no worse when patients are fed early. There may be a trend to quicker hospital discharge in those who get fed earlier. The studies have all been small, and a large multicenter trial would be welcome.

References

1. Gastroenterology. 2007 May;132(5):2022-44.

2. Gut. 1997 Feb;40(2):262-6.

3. Am J Gastroenterol. 2006 Oct;101(10):2379-400.

4. Pancreas. 2010 Oct;39(7):1088-92.

5. Clin Nutr. 2013 Oct;32(5):697-703.

6. Clin Nutr. 2007 Dec;26(6):758-63.

Dr. Paauw is professor of medicine in the division of general internal medicine at the University of Washington, Seattle, and he serves as third-year medical student clerkship director at the University of Washington. Contact Dr. Paauw at dpaauw@uw.edu.

Publications
Topics
Legacy Keywords
acute pancreatitis
Sections

A 60-year-old man presents to the emergency department with nausea and abdominal pain, and is admitted with pancreatitis due to alcohol. In the evening after receiving pain medication, his abdominal pain is diminished but still present. He has an appetite and asks for food.

What do you recommend?

A. Nil per os (NPO) until pain is resolved.

B. NPO until amylase/lipase have normalized.

C. Nasogastric tube placement.

D. Okay to start feeding.

Myth: Treatment of pancreatitis includes early avoidance of food.

 

RTEmagicC_2d37f4d571531c22fc_Paauw_Douglas_S_WA.jpg.jpg
Dr. Douglas S. Paauw

The conventional management of acute pancreatitis involves an NPO regimen until the pain and nausea have resolved.1 This dogma is offered because of the concern that food intake will stimulate pancreatic enzyme release in an already inflamed/injured pancreas.

The approach of NPO and slowly reintroducing feeding after prolonged periods of being without food is associated with pain relapses and increased length of hospitalizations.2 Nasojejunal feedings have become well accepted in patients with severe pancreatitis requiring ICU care.3

Are there data to show that oral feeding of patients with mild pancreatitis causes worse outcomes?

Dr. Niels Teich and colleagues randomized 143 hospitalized patients with mild pancreatitis to eating when they felt ready to (69 patients) vs. a group that were kept NPO until lipase levels returned to normal.4 The patients who started eating when they were ready left the hospital a day earlier than the patients who were fed only when lipase levels normalized (7 days vs. 8 days). There was no difference in abdominal pain between the two groups.

Dr. Maxim Petrov and colleagues looked at whether nasogastric tube feeding was preferable to NPO in patients with mild to moderate pancreatitis.5 In a randomized trial of 35 patients with pancreatitis, 17 received nasogastric feedings within 24 hours of admission, and 18 were NPO. The patients who received early nasogastric feedings had lower pain scores at 72 hours, compared with the NPO group (1 vs. 3 on a visual analog 10-point scale, P = .036). The number of patients who did not require opiates at 48 hours was also significantly less in the nasogastric feeding group (9 vs. 3, P = .024).

I think the most striking difference was in patients’ ability to tolerate oral feeding. Patients in both groups received oral food at an average of 4 days; only 1 of 17 patients in the nasogastric feeding group could not tolerate feeding, compared with 9 of 18 patients in the NPO group.

Dr. Gunilla Eckerwall and colleagues studied the outcome of immediate oral feeding in patients with mild pancreatitis.6 Sixty patients with mild acute pancreatitis, defined by amylase greater than 3 times normal and APACHE scores less than 8, were randomized to either immediate oral feeding (30 patients) or fasting (30 patients). Key outcome measures in the study were amylase, systemic inflammatory response, and length of hospital stay.

There were no differences in amylase levels, labs measuring systemic inflammatory response, or gastrointestinal symptoms between the two groups. The immediate oral feeding group had a significantly shorter length of hospital stay than the fasting group (4 days vs. 6 days, P less than .05).

So, what does all this tell us about feeding patients with acute pancreatitis? For mild to moderate acute pancreatitis, the outcomes appear to be no worse when patients are fed early. There may be a trend to quicker hospital discharge in those who get fed earlier. The studies have all been small, and a large multicenter trial would be welcome.

References

1. Gastroenterology. 2007 May;132(5):2022-44.

2. Gut. 1997 Feb;40(2):262-6.

3. Am J Gastroenterol. 2006 Oct;101(10):2379-400.

4. Pancreas. 2010 Oct;39(7):1088-92.

5. Clin Nutr. 2013 Oct;32(5):697-703.

6. Clin Nutr. 2007 Dec;26(6):758-63.

Dr. Paauw is professor of medicine in the division of general internal medicine at the University of Washington, Seattle, and he serves as third-year medical student clerkship director at the University of Washington. Contact Dr. Paauw at dpaauw@uw.edu.

A 60-year-old man presents to the emergency department with nausea and abdominal pain, and is admitted with pancreatitis due to alcohol. In the evening after receiving pain medication, his abdominal pain is diminished but still present. He has an appetite and asks for food.

What do you recommend?

A. Nil per os (NPO) until pain is resolved.

B. NPO until amylase/lipase have normalized.

C. Nasogastric tube placement.

D. Okay to start feeding.

Myth: Treatment of pancreatitis includes early avoidance of food.

 

RTEmagicC_2d37f4d571531c22fc_Paauw_Douglas_S_WA.jpg.jpg
Dr. Douglas S. Paauw

The conventional management of acute pancreatitis involves an NPO regimen until the pain and nausea have resolved.1 This dogma is offered because of the concern that food intake will stimulate pancreatic enzyme release in an already inflamed/injured pancreas.

The approach of NPO and slowly reintroducing feeding after prolonged periods of being without food is associated with pain relapses and increased length of hospitalizations.2 Nasojejunal feedings have become well accepted in patients with severe pancreatitis requiring ICU care.3

Are there data to show that oral feeding of patients with mild pancreatitis causes worse outcomes?

Dr. Niels Teich and colleagues randomized 143 hospitalized patients with mild pancreatitis to eating when they felt ready to (69 patients) vs. a group that were kept NPO until lipase levels returned to normal.4 The patients who started eating when they were ready left the hospital a day earlier than the patients who were fed only when lipase levels normalized (7 days vs. 8 days). There was no difference in abdominal pain between the two groups.

Dr. Maxim Petrov and colleagues looked at whether nasogastric tube feeding was preferable to NPO in patients with mild to moderate pancreatitis.5 In a randomized trial of 35 patients with pancreatitis, 17 received nasogastric feedings within 24 hours of admission, and 18 were NPO. The patients who received early nasogastric feedings had lower pain scores at 72 hours, compared with the NPO group (1 vs. 3 on a visual analog 10-point scale, P = .036). The number of patients who did not require opiates at 48 hours was also significantly less in the nasogastric feeding group (9 vs. 3, P = .024).

I think the most striking difference was in patients’ ability to tolerate oral feeding. Patients in both groups received oral food at an average of 4 days; only 1 of 17 patients in the nasogastric feeding group could not tolerate feeding, compared with 9 of 18 patients in the NPO group.

Dr. Gunilla Eckerwall and colleagues studied the outcome of immediate oral feeding in patients with mild pancreatitis.6 Sixty patients with mild acute pancreatitis, defined by amylase greater than 3 times normal and APACHE scores less than 8, were randomized to either immediate oral feeding (30 patients) or fasting (30 patients). Key outcome measures in the study were amylase, systemic inflammatory response, and length of hospital stay.

There were no differences in amylase levels, labs measuring systemic inflammatory response, or gastrointestinal symptoms between the two groups. The immediate oral feeding group had a significantly shorter length of hospital stay than the fasting group (4 days vs. 6 days, P less than .05).

So, what does all this tell us about feeding patients with acute pancreatitis? For mild to moderate acute pancreatitis, the outcomes appear to be no worse when patients are fed early. There may be a trend to quicker hospital discharge in those who get fed earlier. The studies have all been small, and a large multicenter trial would be welcome.

References

1. Gastroenterology. 2007 May;132(5):2022-44.

2. Gut. 1997 Feb;40(2):262-6.

3. Am J Gastroenterol. 2006 Oct;101(10):2379-400.

4. Pancreas. 2010 Oct;39(7):1088-92.

5. Clin Nutr. 2013 Oct;32(5):697-703.

6. Clin Nutr. 2007 Dec;26(6):758-63.

Dr. Paauw is professor of medicine in the division of general internal medicine at the University of Washington, Seattle, and he serves as third-year medical student clerkship director at the University of Washington. Contact Dr. Paauw at dpaauw@uw.edu.

Publications
Publications
Topics
Article Type
Display Headline
Myth of the Month: NPO good for people with pancreatitis?
Display Headline
Myth of the Month: NPO good for people with pancreatitis?
Legacy Keywords
acute pancreatitis
Legacy Keywords
acute pancreatitis
Sections
Disallow All Ads